You are on page 1of 176

MUHAMMAD FAISAL

MIT 4th Semester


Al-Barq Campus (VGJW01) Gujranwala
faisalgrw123@gmail.com
Reference MCQ’s For MIDTERM EXAMS
CS408 - HUMAN COMPUTER INTERACTION

Question No: 1 (Marks: 1) - Please choose one


are unintentional while occur through conscious deliberation.

 Slips, mistakes (page 100)


 Errors, slips
 Mistakes, errors
 Mistakes, slips

Question No: 2 (Marks: 1) - Please choose one


What is the main strength of the Problem Space Framework as a model of human problem solving?

 It operates within the constraints of the human processing system (page 91)
 It explains what is involved in insight
 It allows ill-defined problems to be solved
 None of these

Question No: 3 (Marks: 1) - Please choose one


is the process of selecting things to concentrate on, at a point in time, from the
range of possibilities available.

 Perception and recognition


 Attention (page 76)
 Learning
 None of these
Question No: 4 (Marks: 1) - Please choose one
Interpretation inquiry, according to Beyer and Holtzblatt, is based on a master-apprentice model of
learning.

 True
 False (page 176)

Question No: 5 (Marks: 1) - Please choose one


The persona is not an actual user of the product, but is indirectly affected by it and its use refers to
persona

 Primary
 Secondary
 Served (page 159)
 Negative

Question No: 6 (Marks: 1) - Please choose one


is a powerful, multipurpose design tool that helps overcome several problems that
currently plague the development of digital products.

 Scenario
 Persona (page 187)
 Prototype

Question No: 7 (Marks: 1) - Please choose one


The difference between the intentions and allowable actions is the:

 Gulf of Execution (page 103)


 Gulf of Evaluation
 None of these

Question No: 8 (Marks: 1) - Please choose one


Attention is the process of selecting things to concentrate on, at a point in time, from the range of

 Possibilities available (page 76)


 Time Available
 None of these

Question No: 9 (Marks: 1) - Please choose one


The goals of HCI are:

 Usability and User Experience (page 30)


 Learn ability and Comfort
 Tasks and Goals
Question No: 10 (Marks: 1) - Please choose one
WYSIWYG stands for .

 Where you see is where you get


 What you see is what you get (page 36)
 When you see it when you get

Question No: 11 (Marks: 1) - Please choose one


is proportional to the amplitude of the sound.

 Pitch
 Loudness (page 70)
 Timber

Question No: 12 (Marks: 1) - Please choose one


Persona is not context specific, so it can be reused easily.

 True
 False (page 189)

Question No: 13 (Marks: 1) - Please choose one


are individual and isolated regions within display that can be selected by the user to
invoke specific operations.

 Buttons (page 133)


 Pointers
 Menus
 Windows

Question No: 14 (Marks: 1) - Please choose one


User personas that are not primary or secondary are personas.

 Served
 Negative
 Customer
 Supplemental (page 196)

Question No: 15 (Marks: 1) - Please choose one


Which of the following is the comparison of what happened in the world with what we wanted to
happen?

 Action
 Evaluation (page 99)
 Execution
 None of these
Question No: 16 (Marks: 1) - Please choose one
The size and shape of a compact disk, is a type of constraint.

 Physical (page 106)


 Logical
 Cultural
 None of these

Question No: 17 (Marks: 1) - Please choose one


Research can tell you about what, how, many and why in rich, multivariate detail.

 Quantitative
 Qualitative (page 166)
 SME
 None of these

Question No: 18 (Marks: 1) - Please choose one


Which of the given statements correctly defines effectiveness in terms of one of the usability goals?

 It is a very general goal and refers to how good a system at doing what it is suppose to do. (page 31)
 It refers to the way a system supports users in carrying out their tasks.
 It involves protecting the users from dangerous conditions
 It involves protecting the users from undesired situations

Question No: 19 (Marks: 1) - Please choose one


Which of the following is not a secondary color?

 Green
 Orange
 Purple
 Blue (page 63)

Question No: 20 (Marks: 1) - Please choose one


Which of the following device can not be useful for a visually impaired person?

 a typical keyboard
 a typical monitor
 a typical speaker
 a typical processor

Question No: 21 (Marks: 1) - Please choose one


Which of the following requires less cognitive effort than others?

 Listening (page 87)


 Speaking
 Hearing
 None of the given
Question No: 22 (Marks: 1) - Please choose one
We are deficient in our development , not in our development
(respectively).

 Process, Tools (page 23)


 Tools, Process
 Tools, Methodology
 None of these

Question No: 23 (Marks: 1) - Please choose one


Which of the following is not true regarding “cones”?

 A type of receptor in eye more sensitive to light (page 56)


 Different types of cones are sensitive to different wavelengths of light
 Eye has approximately 6 million cones

Question No: 24 (Marks: 1) - Please choose one


Which of the following does not count as one of the advantages of computers?

 Computers are enabling new discoveries


 Leading to efficiencies
 Making our life convenient
 Depending so much on computers may give fatal results (page 9)

Question No: 25 (Marks: 1) - Please choose one


The gulf of execution refers to .

The user’s difficulty in understanding the feedback from the system


The system’s difficulty in presenting information in the output language
The system’s difficulty in converting an input expression into the correct system state transition
The user’s difficulty in formulating and articulating an intention to the system

Question No: 26 (Marks: 1) - Please choose one


Learnability, flexibility and robustness are three main usability principles that can be considered as
general headings for standards and guidelines generation. Which of the following are also high level
usability categories that can guide standards and guidelines generation? (i) Effectiveness (ii)
Efficiency (iii) Fault tolerance (iv) Satisfaction

Select correct option:

(i) & (ii) (page 31)


(i), (ii) & (iv)
(ii) & (iii)
(ii) & (iv)
Question No: 27 (Marks: 1) - Please choose one
Which of the following is not true?

 Utility refers to the functionality of a system


 Usability is concerned with adding complexity to the system (page 31)
 Usability is concerned with making systems easy to use
 Poorly designed computer system can be extremely annoying to users

Question No: 28 (Marks: 1) - Please choose one


The command line interface is used because:

 It is easy to understand
 It is demanded by DOS
 It is offered by UNIX systems
 It is powerful and flexible (page 127)

Question No: 29 (Marks: 1) - Please choose one


Which of the given statements correctly defines effectiveness in terms of one of the usability goals?

 It is a very general goal and refers to how good a system at doing what it is suppose to do. (page 31)
 It refers to the way a system supports users in carrying out their tasks.
 It involves protecting the users from dangerous conditions
 It involves protecting the users from undesired situations

Question No: 30 (Marks: 1) - Please choose one


Which of the following is not one of the primary colors?

 Red
 Yellow
 Blue
 Green (page 63)

Question No: 31 (Marks: 1) - Please choose one


Which of the following is concerned primarily with understanding human behavior and the mental
processes that underlie it?

 Psychology (page 43)


 Sociology
 Statistics
 Computer Science

Question No: 32 (Marks: 1) - Please choose one


HCI deals with:

 Design of interactive system only


 Evaluation of interactive system only
 Implementation of interactive system only
 All of the given choices (page 15)
Question No: 33 (Marks: 1) - Please choose one
Human beings interact with outside world, using their

 input channels
 output channels
 sense of sight
 All of the given (page 54)

Question No: 33 (Marks: 1) - Please choose one


We are deficient in our development , not in our development (respectively).

 Process, Tools (page 23)


 Tools, Process
 Tools, Methodology
 None of these

Question No: 34 (Marks: 1) - Please choose one


Visually impaired persons can interact with outside world using their

 Sense of sight
 Sense of hearing
 Both sense of touch and sense of hearing
 Sense of touch

Question No: 35 (Marks: 1) - Please choose one


is the process of selecting things to concentrate on, at a point in time, from the
range of possibilities available

 Reasoning
 Decision Making
 Attention (page 76)

Question No: 36 (Marks: 1) - Please choose one


refers to the relationship between controls and their effects in the world.

 Visibility
 Affordance
 Mapping (page 107)

Question No: 37 (Marks: 1) - Please choose one


. plays a role to bridge up the gap between the interfaces of machines and
human understanding.

 Human computer Interaction (page 14)


 Software Engineering
 Human Computer Interfaces
Question No: 38 (Marks: 1) - Please choose one
is a very general goal of Usability and refers to how good a system at doing what it is
suppose to do.

 Effectiveness (page 31)


 Efficiency
 Utility

Question No: 39 (Marks: 1) - Please choose one


. is what goes on in out heads when we carry out our everyday activities?

 Cognition (page 47)


 Learnability
 Memeorability

Question No: 40 (Marks: 1) - Please choose one


is the process by which we use the knowledge we have to draw conclusions or
infer something new about the domain of interest.

 Decision Making
 Reasoning (page 89)
 Problem Solving

Question No: 41 (Marks: 1) - Please choose one


research helps us understand the domain, context and constraints of a product in
different, more useful ways than _ research do.

 Qualitative, Quantitative (page 167)


 Quantitative, Qualitative
 None of them

Question No: 42 (Marks: 1) - Please choose one


In an organization individuals may keep their own records, or there may be local gurus. This
statement concerns with .

 Paper work and computer work


 Spatial and temporal organizations
 Organizational memory (page 176)
 None of these

Question No: 43 (Marks: 1) - Please choose one


Using icons on the desktop to represent operations is a type of _ constraint.

 Physical
 Logical
 Cultural (page 106)
 None of these
Question No: 44 (Marks: 1) - Please choose one
is a term used to refer to an attribute of an object that allows people to know how to
use it.

 Visibility
 Affordance (page 105)
 Constraint
 None of these

Question No: 45 (Marks: 1) - Please choose one


minimizes errors.

 Affordance
 Visibility (page 104)
 Constraints
 None of these

Question No: 46 (Marks: 1) - Please choose one


Which of the following is proportional to the amplitude of the sound; the frequency remains constant?

 Pitch
 Loudness (page 70)
 Timber
 None of these

Question No: 47 (Marks: 1) - Please choose one


plays a role to bridge up the gape between the interfaces of machines and human
understanding.

 Human
 Computer
 Human Computer Interaction (page 14)
 None of these

Question No: 48 (Marks: 1) - Please choose one


The persona’s needs are sufficiently unique to require a distinct interface form and behavior is
type of Persona.

 Primary (page 159)


 Secondary
 Supplement
 Negative

Question No: 49 (Marks: 1) - Please choose one


A is usually a collection of icons those are reminiscent of the purpose of the various modes.

 Button
 Pointer
 Palette (page 133)
 Title bar
Question No: 50 (Marks: 1) - Please choose one
The persona is not an actual user of the product, but is indirectly affected by it and its use refers to
persona

 Primary
 Secondary
 Served (page 159)
 Negative

Question No: 51 (Marks: 1) - Please choose one


During phase, usage and workflow patterns discovered through .

 Modeling, analysis (page 159)


 Analysis, modeling
 Testing, modeling
 Testing, Analysis

Question No: 52 (Marks: 1) - Please choose one


Waterfall model is basically a model in which each step must be completed before the
next step can be started:

 Incremental
 Linear (page 149)
 Iterative
 Analytical

Question No: 53 (Marks: 1) - Please choose one


Models are used in design to,

 Generate the design


 Evaluate the design
 Generate and evaluate the design (page 185)
 None of the given

Question No: 54 (Marks: 1) - Please choose one


What is a semantic network?

 A model of long-term memory (page 82)


 A record of our memory of events
 The part of the brain which allows us to remember things
 A mechanism for improving memory

Question No: 55 (Marks: 1) - Please choose one


Which are the most significant senses for the average person when it comes to interacting with a
computer?

 Sight and hearing


 Hearing, touch and smell
 Hearing and touch
 Sight, hearing and touch (page 54)
Question No: 56 (Marks: 1) - Please choose one
refers to the way a system supports users in carrying out their tasks.

 Efficiency (page 31)


 Effectiveness
 Utility

Question No: 57 (Marks: 1) - Please choose one


are GUIs that consists of electronic counterparts to physical objects in the real
world to match the knowledge requirements of users.

 User Interaction Models


 Conceptual Models
 Interface Metaphors (page 51)

Question No: 58 (Marks: 1) - Please choose one


language tends to be grammatical while language is often Ungrammatical.

 Written, spoken (page 87)


 Spoken, written
 None of the given choices

Question No: 59 (Marks: 1) - Please choose one


Aspect gives us hints and tips about using and creating user interface idioms.

 Strategic
 Tactical (page 143)
 None of the given choices

Question No: 60 (Marks: 1) - Please choose one


The Usability Engineering lifecycle was proposed by .

 Deborah Mayhew (page 153)


 Webster
 Barry Boehm
 Hartson

Question No: 61 (Marks: 1) - Please choose one


Ali wants to develop an application that incorporates print, audio video media and software
applications on a single place. Which of the following will help in this regard?

 MS World
 Worldwide Web (page 305)
 Television Newspaper

Question No: 62 (Marks: 1) - Please choose one


Ali is looking at how interface designers went about their work. He identified two different
modes of activity: one is and other is .

 Analytic mode, synthetic mode (page 153)


 Evaluation mode, testing mode
 Analyze mode, feedback mode
 Implementation mode, task mode
Question No: 63 (Marks: 1) - Please choose one
Scenario content and context are derived from information gathered during the
phase and analyzed during the phase (respectively).

 Modeling, Implementation
 Modeling, Research
 Research, Implementation
 Research, Modeling (page 199)

Question No: 64 (Marks: 1) - Please choose one


Ali requires access via a navigational portal relatively infrequently that provides clear, simple
navigational control. Which of the following posture is appropriate in this case?

 Auxiliary
 Transient (page 231)
 Information
 Sovereign

Question No: 65 (Marks: 1) - Please choose one


are dragged down from the title at the top of the screen.

 Pull Down Menus (page 132)


 Main Menus
 Icons
 Buttons

Question No: 66 (Marks: 1) - Please choose one


When you engaged in a conversation you are more attentive what the other person is saying. It is called?

 Focused Attention
 Voluntary Attention (page 78)
 Involuntary attention
 Divided Attention

Question No: 67 (Marks: 1) - Please choose one


Drive a vehicle while holding a conversation with a passenger is the example of .

 Focused Attention
 Voluntary Attention
 Involuntary attention
 Divided Attention (page 78)

Question No: 68 (Marks: 1) - Please choose one


GOMS stands for:

 Goals, operation, methods and selection rules (page 50)


 Goals, objects, models and selection rules
 Goals, operations, methods and state rules
 Goals, operations, models and state rules
Question No: 69 (Marks: 1) - Please choose one
Which of the following is/are the main component(s) of color?

 Hue
 Intensity
 Saturation
 All of the given (page 59)

Question No: 70 (Marks: 1) - Please choose one


represent the user’s expectations of the tangible outcomes of using a specific product.

 Non-user goals
 End goals (page 192)
 Experience goals
 Life goals

Question No: 71 (Marks: 1) - Please choose one


Persona is not context specific, so it can be reused easily.

 True
 False (page 189)

Question No: 72 (Marks: 1) - Please choose one


In an email application configuring a new email server is a key path activity.

 True (page 207)


 False

Question No: 73 (Marks: 1) - Please choose one


are perhaps the least-documented patterns, but they are nonetheless in widespread
use.

 Postural
 Structural (page 217)
 Behavioral
 Mnemonic

Question No: 74 (Marks: 1) - Please choose one


The way people carry out an activity in the real world is same as how it may be observed in the
laboratory

 True
 False (page 256)

Question No: 75 (Marks: 1) - Please choose one


Engineering a user interface involves a quantitative engineering style of working in which
measurements are used to judge the quality of interface. Hence is appropriate.

 Usability testing (page 264)


 Field study
 Predictive evaluation
 DECIDE framework
Question No: 76 (Marks: 1) - Please choose one
Exploring how children talk together in order to see if an innovative new groupware product would
help them to be more engaged would probably be better informed by a .

 Usability testing
 Field study (page 264)
 Predictive evaluation
 DECIDE framework

Question No: 77 (Marks: 1) - Please choose one


Beneath the surface is the of the site: the placement of buttons, tabs, photos, and
blocks of text.

 Surface
 Structure
 Skeleton (page 306)
 Scope

Question No: 78 (Marks: 1) - Please choose one


Roughly percent of the male population has some degree of color blindness.

 10 (page 358)
 20
 30
 40

Question No: 79 (Marks: 1) - Please choose one


are individual and isolated regions within display that can be selected by the user to
invoke specific operations.

 Buttons (page 133)


 Pointers
 Menus
 Windows

Question No: 80 (Marks: 1) - Please choose one


The function came into being as the result of the implementation model for undo

 Redo (page 314)


 Undo
 Repeat
 Delete

Question No: 81 (Marks: 1) - Please choose one


If the user wants to place the document somewhere in the file system hierarchy, he can
request this function from the menu.

 Explicitly (page 327)


 Implicitly
 Habitually
 Properly
Question No: 82 (Marks: 1) - Please choose one
Design of user interfaces does not entirely aesthetic concerns, but rather it places
such' concerns within the constraints of a framework.

 Include, Functional
 Exclude, Functional (page 343)
 Include, Non-Functional
 Exclude, Non-Functional

Question No: 83 (Marks: 1) - Please choose one


When developing , plan to keep them short, straightforward and avoid asking too
many.

 Videos
 Documentation
 Interview CS408 Questions (page 371)
 Contextual enquiry

Question No: 84 (Marks: 1) - Please choose one


Conventional wisdom says that tell the user when he has made some mistake.

 Program crash
 System stuck
 Error messages (page 382)
 Metadata

Question No: 85 (Marks: 1) - Please choose one


People frequently counter the idea of feedback with arguments that users don't like it

 Audible (page 393)


 Video
 Walkthroughs
 CS408 Questions

Question No: 86 (Marks: 1) - Please choose one


environments are environments that are user and context aware.

 Non-attentive
 Visual
 Sensing
 Attentive (page 418)

Question No: 87 (Marks: 1) - Please choose one


User personas that are not primary or secondary are personas.

 Served
 Negative
 Customer
 Supplemental (page 196)
Question No: 88 (Marks: 1) - Please choose one
Which of the following is the comparison of what happened in the world with what we wanted to
happen?

 Action
 Evaluation (page 99)
 Execution
 None of these

Question No: 89 (Marks: 1) - Please choose one


The size and shape of a compact disk, is a type of constraint.

 Physical (page 106)


 Logical
 Cultural
 None of these

Question No: 90 (Marks: 1) - Please choose one


research can tell you about what, how, many and why in rich, multivariate detail.

 Quantitative
 Qualitative (page 166)
 SME
 None of these

Question No: 91 (Marks: 1) - Please choose one


is the least technical way of collecting data, but it can be difficult and tiring to write
and observe at the same time.

 Audio recording
 Taking notes (page 365)
 Observation
 Video

Question No: 92 (Marks: 1) - Please choose one


Panes, frames and other containers on screen is a kind of

 Functional and data elements (page 206)


 View
 Input method
 None of the given

Question No: 93 (Marks: 1) - Please choose one


is particularly useful early in design. It is excellent technique to use with the
prototype, because it provides a wealth of diagnostic information.

 Co-discovery
 Active intervention (page 276)
 Splendid research
 None of the given
Question No: 94 (Marks: 1) - Please choose one
Information sites with daily-updated information will naturally attract users more
than a monthly-updated site.

 Repeat (page 229)


 Infrequent
 Nonuser
 None of the given

Question No: 95 (Marks: 1) - Please choose one


Evaluations done during design to check that product continues to meet users’ needs are known as
evaluation.

 Formative (page 258)


 Summative
 Relative
 None of the given

Question No: 96 (Marks: 1) - Please choose one


frequent your primary personas access the site, transient a stance
the site needs to take (respectively).

 More, Less
 Less, more (page 230)
 Less, less
 None of the given

Question No: 97 (Marks: 1) - Please choose one


In an organization individuals may keep their own records, or there may be local gurus. This
statement concerns with .

 Paper work and computer work


 Spatial and temporal organizations
 Organizational memory (page 176)
 None of these

Question No: 98 (Marks: 1) - Please choose one


Currently many common environments for interactive computing are examples of the
interface style, often simply called windowing systems.

 Three-dimensional
 WIMP (page 129)
 Point and click
 None of these

Question No: 99 (Marks: 1) - Please choose one


Using icons on the desktop to represent operations is a type of constraint.

 Physical
 Logical
 Cultural (page 106)
 None of these
Question No: 100 (Marks: 1) - Please choose one
is a term used to refer to an attribute of an object that allows people to know how to
use it.

 Visibility
 Affordance (page 105)
 Constraint
 None of these

Question No: 101 (Marks: 1) - Please choose one


minimizes errors.

 Affordance
 Visibility (page 104)
 Constraints
 None of these

Question No: 102 (Marks: 1) - Please choose one


Which of the following is proportional to the amplitude of the sound; the frequency remains
constant?

 Pitch
 Loudness (page 70)
 Timber
 None of these

Question No: 103 (Marks: 1) - Please choose one


plays a role to bridge up the gape between the interfaces of machines and human
understanding.

 Human
 Computer
 Human Computer Interaction (page 14)
 None of these

Question No: 104 (Marks: 1) - Please choose one


The persona’s needs are sufficiently unique to require a distinct interface form and behavior is
type of Persona.

 Primary (page 159)


 Secondary
 Supplement
 Negative

Question No: 105 (Marks: 1) - Please choose one


A is usually a collection of icons those are reminiscent of the purpose of the various
modes.

 Button
 Pointer
 Palette (page 133)
 Title bar
Question No: 106 (Marks: 1) - Please choose one
The persona is not an actual user of the product, but is indirectly affected by it and its use refers to
persona

 Primary
 Secondary
 Served (page 159)
 Negative

Question No: 107 (Marks: 1) - Please choose one


During phase, usage and workflow patterns discovered through .

 Modeling, analysis (page 159)


 Analysis, modeling
 Testing, modeling
 Testing, Analysis

Question No: 108 (Marks: 1) - Please choose one


The Usability Engineering lifecycle was proposed by .

 Deborah Mayhew (page 153)


 Webster
 Barry Boehm
 Hartson

Question No: 109 (Marks: 1) - Please choose one


Waterfall model is basically a model in which each step must be completed before the
next step can be started

 Incremental
 Linear (page 149)
 Iterative
 Analytical

Question No: 110 (Marks: 1) - Please choose one


Models are used in design to

 Generate the design


 Evaluate the design
 Generate and evaluate the design (page 185)
 None of the given

Question No: 111 (Marks: 1) - Please choose one


What is a semantic network?

 A model of long-term memory (page 82)


 A record of our memory of events
 The part of the brain which allows us to remember things
 A mechanism for improving memory
Question No: 112 (Marks: 1) - Please choose one
Which are the most significant senses for the average person when it comes to interacting with a
computer?

 Sight and hearing


 Hearing, touch and smell
 Hearing and touch
 Sight, hearing and touch (page 54)

Question No: 113 (Marks: 1) - Please choose one


In an organization individuals may keep their own records, or there may be local gurus. This
statement concerns with .

 Paper work and computer work


 Spatial and temporal organizations
 Organizational memory (page 176)
 None of these

Question No: 115 (Marks: 1) - Please choose one


Currently many common environments for interactive computing are examples of the
interface style, often simply called windowing systems.

 Three-dimensional
 WIMP (page 129)
 Point and click
 None of these

Question No: 116 (Marks: 1) - Please choose one


Using icons on the desktop to represent operations is a type of constraint.

 Physical
 Logical
 Cultural (page 106)
 None of these

Question No: 117 (Marks: 1) - Please choose one


is a term used to refer to an attribute of an object that allows people to know how to
use it.

 Visibility
 Affordance (page 105)
 Constraint
 None of these

Question No: 118 (Marks: 1) - Please choose one


minimizes errors.

 Affordance
 Visibility (page 104)
 Constraints
 None of these
Question No: 119 (Marks: 1) - Please choose one
Which of the following is proportional to the amplitude of the sound; the frequency remains
constant?

 Pitch
 Loudness (page 70)
 Timber
 None of these

Question No: 120 (Marks: 1) - Please choose one


plays a role to bridge up the gape between the interfaces of machines and human
understanding.

 Human
 Computer
 Human Computer Interaction (page 14)
 None of these

Question No: 121 (Marks: 1) - Please choose one


The persona’s needs are sufficiently unique to require a distinct interface form and behavior is
type of Persona.

 Primary (page 159)


 Secondary
 Supplement
 Negative

Question No: 122 (Marks: 1) - Please choose one


A is usually a collection of icons those are reminiscent of the purpose of the various
modes.

 Button
 Pointer
 Palette (page 133)
 Title bar

Question No: 123 (Marks: 1) - Please choose one


The persona is not an actual user of the product, but is indirectly affected by it and its use refers to
persona

 Primary
 Secondary
 Served (page 159)
 Negative

Question No: 124 (Marks: 1) - Please choose one


During phase, usage and workflow patterns discovered through .

 Modeling, analysis (page 159)


 Analysis, modeling
 Testing, modeling
 Testing, Analysis
Question No: 125 (Marks: 1) - Please choose one
The Usability Engineering lifecycle was proposed by .

 Deborah Mayhew (page 153)


 Webster
 Barry Boehm
 Hartson

Question No: 126 (Marks: 1) - Please choose one


Waterfall model is basically a model in which each step must be completed before the
next step can be started

 Incremental
 Linear (page 149)
 Iterative
 Analytical

Question No: 127 (Marks: 1) - Please choose one


Models are used in design to

 Generate the design


 Evaluate the design
 Generate and evaluate the design (page 185)
 None of the given

Question No: 128 (Marks: 1) - Please choose one


What is a semantic network?

 A model of long-term memory (page 82)


 A record of our memory of events
 The part of the brain which allows us to remember things
 A mechanism for improving memory

Question No: 129 (Marks: 1) - Please choose one


Which are the most significant senses for the average person when it comes to interacting with a
computer?

 Sight and hearing


 Hearing, touch and smell
 Hearing and touch
 Sight, hearing and touch (page 54)

Question No: 130 (Marks: 1) - Please choose one


is a term used to refer to an attribute of an object that allows people to know how to
use it.

 Visibility
 Affordance (page 105)
 Constraint
 None of these
Question No: 131 (Marks: 1) - Please choose one
What is a semantic network?

 A model of long-term memory (page 82)


 A record of our memory of events
 The part of the brain which allows us to remember things
 A mechanism for improving memory

Question No: 132 (Marks: 1) - Please choose one


You can load a VCR tape the right way because of .

 Physical constraints (page 106)


 Logical constraints
 Cultural constraints
 None of these

Question No: 133 (Marks: 1) - Please choose one


A mouse button invites pushing by the way it is physically constrained in its plastic shell, is an
example of Design Principle.

 Visibility
 Affordance (page 105)
 Mapping
 None of these

Question No: 134 (Marks: 1) - Please choose one


aspect gives us hints and tips about using and creating user interface idioms, like dialog
boxes and pushbuttons.

A. Efficient. B. Strategic. C. Tactical. D. Reliable.

Select correct option:

 Only A
 Only C (page 143)
 A and C
 B and D

Question No: 135 (Marks: 1) - Please choose one


"Mistakes" and "Slips" are two forms of:

 Errors (page 100)


 Goals
 Evaluation
 Objectives
Question No: 136 (Marks: 1) - Please choose one
Ergonomics is also called .

 Assembly
 Human Factors (page 43)
 Both
 None of the Given

Question No: 138 (Marks: 1) - Please choose one


User and system has its own unique language, the language used by the system is called

 Core (page 123)


 Task
 Both
 None of the above

Question No: 139 (Marks: 1) - Please choose one


Choice of operations and services are offered on the screen through

 Pointers
 Toolbars
 Menus (page 131)
 None of the given

Question No: 140 (Marks: 1) - Please choose one


You are given 4 cards displaying 4, E, 7 and K respectively, and you are told that if a card has a
vowel on one side it has an even number on the other. Which cards would you have to turn over to
check whether this is the case?

 E and 4
 E and 7
 E, 4 and 7
 4, E, 7 and K

Question No: 141 (Marks: 1) - Please choose one


Which of the following is true regarding "Visibility"?

The more visible functions are, the more likely users will be able to know what to do next. (page 104)
The more visible functions are, the less likely users will be able to know what to do next.
Functions visibility and user knowing of what to do next, are not related to each other
None of the given

Question No: 142 (Marks: 1) - Please choose one


What will be the gulf of execution if the user is able to formulate and perform the actions easily?

 Smaller (page 122)


 Greater
 Balanced
 None of the Given
Question No: 143 (Marks: 1) - Please choose one
Certain factors that help us to interact with graphical based systems is .

 Indirect manipulation
 Direct manipulation (page 36,137)
 Both
 None of the given

Question No: 144 (Marks: 1) - Please choose one


Which interface system is based on the question/answer dialogue?

 Command Line Interfaces


 Query interfaces (page 128)
 Menus
 Natural Language Interfaces

Question No: 145 (Marks: 1) - Please choose one


Taking some real world concept to perform different tasks that represent some idea, inside the
computer is type of .

 Direct Manipulation (page 125,137)


 Desktop metaphors
 Central processing unit
 None of the given

Question No: 146 (Marks: 1) - Please choose one


Example of form fill interface is .

 A dialog box
 Pop up menus
 Data entry form on some university web
 All of the given

Question No: 147 (Marks: 1) - Please choose one


Building things from user's perspective is called _ .

 Functionality
 Usability (page 31)
 Portability
 None of the given

Question No: 148 (Marks: 1) - Please choose one


A small picture that is used to represent a closed window is said to be a/an:

 Menu
 Toolbar
 status bar
 icon (page 131)
Question No: 149 (Marks: 1) - Please choose one
Form-filling interfaces are used for:

 Data entry (page 128)


 Data integration
 Data manipulation
 Data definition

Question No: 150 (Marks: 1) - Please choose one


is an early release of a product to a few users.

Surveys
Beta test (page 271)
Focus

Question No: 151 (Marks: 1) - Please choose one


has a goal of assessing whether the product works according to its specifications.

Trunk Test
Quality assurance (page 270)

Question No: 152 (Marks: 1) - Please choose one


In usability test, which is a technique from given below options, in which you have two participants
work together to perform the tasks.

Active Intervention
Co-Discovery (page 276)
Trunk test

Question No: 153 (Marks: 1) - Please choose one


What site is this”, Statement represents .

Folder
Site ID (page 296)
Scroll bar

Question No: 154 (Marks: 1) - Please choose one


In DECIDE frame work comes just after choosing the evaluation paradigm and
techniques.

 Deciding about ethical issues (page 265)


 Exploring the questions
 Identifying the practical issues
 None of the given
Question No: 155 (Marks: 1) - Please choose one
Formal methods are used to represent:

 Architectural aspects of software systems only


 Procedural aspects of software systems only
 Both architectural and procedural aspects of software systems (page 21)
 None of the given

Question No: 156 (Marks: 1) - Please choose one


The colors, which are directly opposite to each other are said to be:

 Complementary colors (page 64)


 Analogous colors
 Primary colors
 Secondary colors

Question No: 157 (Marks: 1) - Please choose one


Which of the following is true about Short-Term memory?

 Short-term memory has a limited capacity. (page 80)


 Short-term memory has an unlimited capacity.
 Short-term memory has no capacity.
 Short-term memory has large but limited capacity.

Question No: 158 (Marks: 1) - Please choose one


Which of the following is not one of the goals of HCI?

 To produce usable systems


 To produce safe systems
 To produce non-functional systems (page 30)
 To improve effectiveness of the systems

Question No: 159 (Marks: 1) - Please choose one


According to Juran, the fitness for purpose or use, is said to be

 Usability
 Efficiency
 Reliability
 Quality (page 39)

Question No: 160 (Marks: 1) - Please choose one


Which of the given statements correctly defines efficiency in terms of one of the usability goals?

 It is a very general goal and refers to how good a system at doing what it is suppose to do.
 It refers to the way a system supports users in carrying out their tasks. (page 31)
 It involves protecting the users from dangerous conditions
 It involves protecting the users from undesired situations
Over a short period of time, we find it easier to remember the string of numbers " 404 894 6743"
because .

 Numbers are easier to remember than arbitrary characters.


 The grouping of the numbers is significant
 Ten numbers is not that many to have to remember from working memory.
 None of these

Question No: 162 (Marks: 1) - Please choose one


Which of the following is considered as the most complex species?

 Humans (page 18)


 Computers
 Animals
 Birds

Question No: 163 (Marks: 1) - Please choose one


Which of the following is the statement given by "Novatech" as a result of a survey on the
embarrassing experiences of computers?

 One in every four computers has been physically attacked by its owner
 Every computer can be saved from any type of physical attacks
 Every computer is attacked physically one in its life time
 One in every four computers has been physically attacked by some outside source (page 24)

Question No: 164 (Marks: 1) - Please choose one


Which of the following is/are goal(s) of usability?

 Effectiveness
 Efficiency
 Safety
 All of the given (page 31)

Question No: 165 (Marks: 1) - Please choose one


XYZ Ltd is well reputed software house; they make a significant investment in building -------------
------ that encourages greater customer loyalty.

 Visual Symbols
 Grouping
 Brand Equity (page 353)
 Harmony

Question No: 166 (Marks: 1) - Please choose one


What are the three types of reasoning?

 Deductive, Productive and Inductive (page 89)


 Adductive, Inductive and Deductive
 Inductive, Adductive and Reproductive
 Productive, Reproductive and Deductive
is a usability test in which a member of the test team
sits in the room with the participant and actively probes the
participant's understanding of whatever is being tested.

 Co discovery
 Active intervention (page 276)
 Trunk test

Question No: 168 (Marks: 1) - Please choose one


Totality of characteristics of an entity that bear on its ability to satisfy
stated and implied needs are called .

 Quality (page 40)


 Effectiveness
 Standard
 Quantity
The persona is not an actual user of the product, but is indirectly affected by it and its use
refers to persona

Primary

Secondary

Served (page 159)

Negative

The difference between the intentions and allowable actions is the:

Gulf of Execution (page 103)

Gulf of Evaluation 

None of these

is proportional to the amplitude of the sound. Pitch

Loudness (page 70)

Timber

User personas that are not primary or secondary are personas.

Served

Negative

Customer
Solved with refrence by Salman Abid

Supplemental (page 196)

Which of the following is the comparison of what happened in the world with what we wanted to
happen?

Action

Execution

Evaluation (page 99) 

None of these

Which of the following requires less cognitive effort than others?

Listening (page 87)

Speaking

Hearing

None of the given

(Please choose one The command line interface is used because: It is

easy to understand

It is demanded by DOS

It is offered by UNIX systems

It is powerful and flexible (page 127) 

Which of the given statements correctly defines effectiveness in terms of one of the
usability goals?

It refers to the way a system supports users in carrying out their tasks. 
Solved with refrence by Salman Abid

It is a very general goal and refers to how good a system at doing what it
is suppose to do. (page 31) 

It involves protecting the users from dangerous conditions It

involves protecting the users from undesired situations

refers to the relationship between controls and their effects in the world. Visibility

Affordance

Mapping (page 107) 

is a very general goal of Usability and refers to how good a system at doing what
it is supposed to do.

Effectiveness (page 31)

Efficiency

Utility

Which of the following is proportional to the amplitude of the sound; the frequency remains
constant?

Pitch

Loudness (page 70)

Timber

None of these

Please choose one GOMS stands for:

Goals, operation, methods and selection rules (page 50)

Goals, objects, models and selection rules 

Goals, operations, methods and state rules


Goals, operations, models and state rules

User personas that are not primary or secondary are personas.

Served

Negative

Customer

Supplemental (page 196)

Few persons are talking to each other in a room; In another room some one was working and
go to get some conversation of those person is the example of

. Null attention
Voluntary attention
Involuntary attention 79
Revolution attention

rely on learned conventions, like the use of red for warning, the use of
certain
kinds of signals for danger, and the use of the smiley face to represent happy emotions. Physical
constraints
Logical constraints
Culture constraints (pg 106)
Emotional constraints

refers to how good a system at doing what it is supposed to do.


Safety
Usability
Efficiency
Effectiveness (pg 31)
Model is famous for risk
analysis. Waterfall
Spiral (pg 150)
RAD
Subjective

MCQ for FINALTERM EXAMINATION


CS408- Human Computer Interaction

Question No: 1 ( Marks: 1 ) - Please choose one


Persona is not context specific, so it can be reused easily.

► True

► False

Question No: 2 ( Marks: 1 ) - Please choose one


In an email application configuring a new email server is a key path activity.
► True

► False

Question No: 3 ( Marks: 1 ) - Please choose one


_____________ are perhaps the least-documented patterns, but they are
nonetheless in widespread use.

► Postural

► Structural

► Behavioral

► Mnemonic

Question No: 4 ( Marks: 1 ) - Please choose one


The way people carry out an activity in the real world is same as how it may be
observed in the laboratory
► True

► False

Question No: 5 ( Marks: 1 ) - Please choose one


Engineering a user interface involves a quantitative engineering style of working
in which measurements are used to judge the quality of interface. Hence
____________ is appropriate.

► Usability testing

► Field study

Come and Join Us at www.vustudents.ning.com


► Predictive evaluation

► DECIDE framework

Question No: 6 ( Marks: 1 ) - Please choose one


Exploring how children talk together in order to see if an innovative new
groupware product would help them to be more engaged would probably be
better informed by a ______________.

► Usability testing

► Field study

► Predictive evaluation

► DECIDE framework

Question No: 7 ( Marks: 1 ) - Please choose one


Beneath the surface is the _____________ of the site: the placement of buttons,
tabs, photos, and blocks of text.

► Surface

► Structure

► Skeleton

► Scope

Question No: 8 ( Marks: 1 ) - Please choose one


Roughly _____________ percent of the male population has some degree of color
blindness.

► 10
► 20
► 30
► 40

Question No: 9 ( Marks: 1 ) - Please choose one


__________ are individual and isolated regions within display that can be
selected by the user to invoke specific operations.

Come and Join Us at www.vustudents.ning.com


► Buttons

► Pointers

► Menus

► Windows

Question No: 10 ( Marks: 1 ) - Please choose one


The __________ function came into being as the result of the implementation model
for undo

► Redo

► Undo

► Repeat

► Delete

Question No: 11 ( Marks: 1 ) - Please choose one


If the user wants to _________ place the document somewhere in the file system
hierarchy, he can request this function from the menu.

► Explicitly

► Implicitly

► Habitually

► Properly

Question No: 12 ( Marks: 1 ) - Please choose one


Design of user interfaces does not entirely ___________ aesthetic concerns, but
rather it places such' concerns within the constraints of a _____________
framework.

► Include, Functional

► Exclude, Functional

► Include, Non-Functional

Come and Join Us at www.vustudents.ning.com


► Exclude, Non-Functional

Question No: 13 ( Marks: 1 ) - Please choose one


When developing ____________, plan to keep them short, straightforward and
avoid asking too many.

► Videos

► Documentation

► Interview questions

► Contextual enquiry

Question No: 14 ( Marks: 1 ) - Please choose one


Conventional wisdom says that ________ tell the user when he has made some
mistake.

► Program crash

► System stuck

► Error messages

► Metadata

Question No: 15 ( Marks: 1 ) - Please choose one


People frequently counter the idea of ________feedback with arguments that
users don't like it

► Audible

► Video

► Walkthroughs

► Questions

Question No: 16 ( Marks: 1 ) - Please choose one


___________environments are environments that are user and context aware.

Come and Join Us at www.vustudents.ning.com


► Non-attentive

► Visual

► Sensing
► Attentive

Question No: 17 ( Marks: 1 ) - Please choose one


User personas that are not primary or secondary are ____________ personas.

► Served

► Negative

► Customer

► Supplemental

Question No: 18 ( Marks: 1 ) - Please choose one


Which of the following is the comparison of what happened in the world with
what we wanted to happen?

► Action
► Evaluation
► Execution
► None of these

Question No: 19 ( Marks: 1 ) - Please choose one


The size and shape of a compact disk, is a type of _____________ constraint.
► Physical
► Logical
► Cultural
► None of these

Question No: 20 ( Marks: 1 ) - Please choose one


_____________ Research can tell you about what, how, many and why in rich,
multivariate detail.

► Quantitative

► Qualitative

Come and Join Us at www.vustudents.ning.com


► SME

► None of these

Question No: 21 ( Marks: 1 ) - Please choose one


_____________is the least technical way of collecting data, but it can be
difficult and tiring to write and observe at the same time.

► Audio recording.

► Taking notes.

► Observation
► Video

Question No: 22 ( Marks: 1 ) - Please choose one


Panes, frames and other containers on screen is a kind of _____________
► Functional and data elements

► View

► Input method

► None of the given

Question No: 23 ( Marks: 1 ) - Please choose one


____________ is particularly useful early in design. It is excellent technique to
use with the prototype, because it provides a wealth of diagnostic information.

► Co-discovery

► Active intervention

► Splendid research

► None of the given

Question No: 24 ( Marks: 1 ) - Please choose one


Information sites with daily-updated information will naturally attract
____________ users more than a monthly-updated site.

► Repeat

► Infrequent

Come and Join Us at www.vustudents.ning.com


► Nonuser

► None of the given

Question No: 25 ( Marks: 1 ) - Please choose one


Evaluations done during design to check that product continues to meet
users’ needs are known as _____________ evaluation.
► Formative

► Summative

► Relative

► None of the given

Question No: 26 ( Marks: 1 ) - Please choose one


____________ frequent your primary personas access the site, _____________
transient a stance the site needs to take (respectively).

► More, Less

► Less, more

► Less, less

► None of the given

Question No: 10 ( Marks: 1 ) - Please choose one


___________ suggests identifying goals and questions first before selecting
techniques for the study
► RVM model
► DECIDE framework
► Usability testing
► Field study
Question No: 3 ( Marks: 1 ) - Please choose one
______________ applications are great platforms for creating an
environment rich, in visual feedback for the user.
► Sovereign
► Transient
► Auxiliary
► Daemonic

Question No: 4 ( Marks: 1 ) - Please choose one


___________ is the process of selecting things to concentrate on, at a point in
time, from the range of possibilities available.

Come and Join Us at www.vustudents.ning.com


► Perception and recognition
► Attention
► Learning
► None of these
Question No: 1 ( Marks: 1 ) - Please choose one
__________ is like the building name for a website.
► Site ID
► Navigation
► Section
► None of the given

Question No: 3 ( Marks: 1 ) - Please choose one


__________ is the extra work that satisfies the needs to achieve our objective.
► Evaluation
► Excise
► Testing
► None of the given

Question No: 4 ( Marks: 1 ) - Please choose one


Panes, frames and other containers on screen is a kind of _____________
► Functional and data elements
► View
► Input method
► None of the given
Question No: 5 ( Marks: 1 ) - Please choose one
Which of the following is least likely to be revealed by a paper prototype?
► Your users don’t know the term algorithm
► Toolbar buttons are too small to press.
► The Help menu isn’t in the right place.
► None of the given

Question No: 6 ( Marks: 1 ) - Please choose one


_____________is the least technical way of collecting data, but it can be
difficult and tiring to write and observe at the same time.
► Audio recording.
► Taking notes.
► Observation
► Video

Question No: 7 ( Marks: 1 ) - Please choose one


Every page within a site should contain a brief _________ that accurately
describes the content of the page.
► Site ID.
► Header
► Local Navigation.

Come and Join Us at www.vustudents.ning.com


► Search button

Question No: 8 ( Marks: 1 ) - Please choose one


______________ applications are great platforms for creating an
environment rich, in visual feedback for the user.
► Sovereign
► Transient
► Auxiliary
► Daemonic

Question No: 9 ( Marks: 1 ) - Please choose one


What is the fifth step in defining the requirements with respect to persona-
based design?
► Brainstorming.
► Identifying persona expectations.
► Constructing the context scenario.
► Identifying needs

Question No: 10 ( Marks: 1 ) - Please choose one

You can load a VCR tape the right way because of _____________.
► Physical constraints
► Logical constraints
► Cultural constraints
► None of these

Question No: 11 ( Marks: 1 ) - Please choose one


____________ are unintentional while _____________ occur through conscious
deliberation.
► Slips, mistakes
► Errors, slips
► Mistakes, errors
► Mistakes, slips

Question No: 12 ( Marks: 1 ) - Please choose one


What is the main strength of the Problem Space Framework as a model of
human problem solving?
► It operates within the constraints of the human processing system
► None of these
► It explains what is involved in insight
► It allows ill-defined problems to be solved

Question No: 13 ( Marks: 1 ) - Please choose one


___________ is the process of selecting things to concentrate on, at a point in
time, from the range of possibilities available.

Come and Join Us at www.vustudents.ning.com


► Perception and recognition
► Attention
► Learning
► None of these

Question No: 14 ( Marks: 1 ) - Please choose one


Over a short period of time, we find it easier to remember the string of numbers
“404 894 6743″ because
► Numbers are easier to remember than arbitrary characters.
► The grouping of the numbers is significant
► Ten numbers is not that many to have to remember from working memory.
► None of these

Question No: 16 ( Marks: 1 ) - Please choose one


___________ involves watching and listening to users.

► Observation
► Evaluation
► Qualitative research
► Interaction
Question No: 17 ( Marks: 1 ) - Please choose one

The name of the document should be shown on the application’s__________.


A. Menu bar
B. Title bar
C. Title bar and menu bar
D. Not Title bar and not menu bar

Question No: 18 ( Marks: 1 ) - Please choose one

The established standard suite of _________ for most applications consists of the Save
As dialog, the Save Changes dialog, and the Open File dialog.
► OS management
► File management
► Data management
► Application management

Question No: 19 ( Marks: 1 ) - Please choose one


_________ is the remarkable facility that lets us reverse a previous action.
► Redo
► Undo
► Repeat
► Delete

Question No: 20 ( Marks: 1 ) - Please choose one

Come and Join Us at www.vustudents.ning.com


Explanatory undo is, generally, a much more pleasant feature than ___________.
► Single Undo
► Multiple undo
► Blind undo
► Incremental Undo

Question No: 21 ( Marks: 1 ) - Please choose one


Browser Title always contains the word ‘_______’.
► Home
► Default
► Error
► Browser

Question No: 22 ( Marks: 1 ) - Please choose one


Interpretation inquiry, according to Beyer and Holtzblatt, is based on a master-
apprentice model of learning.
► True
► False
Question No: 25 ( Marks: 1 ) - Please choose one
____________ are the street signs of the web.

► Site IDs
► Home pages
► Page Names
► Sections

Question No: 26 ( Marks: 1 ) - Please choose one


______________ is not like other pages; it has different burdens to bear, different
promises to keep.

► Homepage
► Form
► Navigation bar
► Sections

Question No: 28 ( Marks: 1 ) - Please choose one


Number of keystrokes is ____________ work.
► Logical
► Mnemonic
► Physical
► Structural

Question No: 29 ( Marks: 1 ) - Please choose one


Recalling password is ____________ work.

Come and Join Us at www.vustudents.ning.com


► Logical
► Mnemonic
► Physical
► Structural

Question No: 30 ( Marks: 1 ) - Please choose one

The persona is not an actual user of the product, but is indirectly affected by it and
its use refers to _______ persona

► Primary
► Secondary
► Served
► Negative

Question No: 31 ( Marks: 1 ) - Please choose one


_________ is a powerful, multipurpose design tool that helps overcome
several problems that currently plague the development of digital products.
¾ Scenario
¾ Persona
¾ Prototype
¾ none

Question No: 32 ( Marks: 1 ) - Please choose one


The difference between the intentions and allowable actions is the :
¾ Gulf of Execution
¾ Gulf of Evaluation
¾ None of these

Question No: 33 ( Marks: 1 ) - Please choose one


Attention is the process of selecting things to concentrate on, at a point
in time, from the ran o
¾ Possibilities available
¾ Time Available
¾ None of these

Question No: 34 ( Marks: 1 ) - Please choose one


The goals of HCI are :
¾ Usability and User Experience
¾ Learn ability and Comfort
¾ Tasks and Goals.

Question No: 35 ( Marks: 1 ) - Please choose one


While there can be wide variations in where and how you conduct a usability

Come and Join Us at www.vustudents.ning.com


test, every usability test shares:
¾ Three Characteristics
¾ Five Characteristics
¾ Seven Characteristics

Question No: 36 ( Marks: 1 ) - Please choose one


WYSIWYG stands for ____________.
¾ Where you see is where you get
¾ What you see is what you get
¾ When you see it when you get

Question No: 37 ( Marks: 1 ) - Please choose one


____________ is proportional to the amplitude of the sound.
¾ Pitch
¾ Loudness
¾ Timber

Question No: 49 ( Marks: 1 ) - Please choose one

If the user wants to _________ place the document somewhere in the file system
hierarchy, he can request this function from the menu.

► Explicitly
► Implicitly
► Habitually
► Properly

Question No: 18 ( Marks: 1 ) - Please choose one


Which of the following is the comparison of what happened in the world with
what we wanted to happen?

► Action
► Evaluation
► Execution
► None of these

Question No: 25 ( Marks: 1 ) - Please choose one


Evaluations done during design to check that product continues to meet
users’ needs are known as _____________ evaluation.

► Formative
► Summative
► Relative
► None of the given

Which of the given statements correctly defines effectiveness in terms of one

Come and Join Us at www.vustudents.ning.com


of the usability goals?

¾ It is a very general goal and refers to how good a system at doing what it
is suppose to do.
¾ It refers to the way a system supports users in carrying out their tasks.
¾ It involves protecting the users from dangerous conditions
¾ It involves protecting the users from undesired situations

Question # 2 of 10 (Start time: 07:45:44 PM ) Total Marks: 1


Which of the following is not a secondary color?

Green Orange Purple Blue

Question # 3 of 10 ( Start time: 07:46:45 PM ) Total Marks: 1


Which of the following device can not be useful for a visually impaired person?

¾ a typical keyboard
¾ a typical monitor
¾ a typical speaker
¾ a typical processor

Question # 4 of 10 ( Start time: 07:48:24 PM ) Total Marks: 1


Which of the following requires less cognitive effort than others?

¾ Listening
¾ speaking
¾ hearing
¾ None of the given

Question # 5 of 10 ( Start time: 07:49:20 PM ) Total Marks: 1


We are deficient in our development _________, not in our development
__________ (respectively).

¾ Process, Tools
¾ Tools, Process
¾ Tools, Methodology
¾ None of these

Question # 6 of 10 ( Start time: 07:50:34 PM ) Total Marks: 1


Which of the following is not true regarding “cones”?

¾ a type of receptor in eye more sensitive to light


¾ different types of cones are sensitive to different wavelengths of light
¾ eye has approximately 6 million cones

Question # 7 of 10 ( Start time: 07:51:35 PM ) Total Marks: 1

Come and Join Us at www.vustudents.ning.com


Which of the following does not count as one of the advantages of computers?

¾ Computers are enabling new discoveries


¾ Leading to efficiencies
¾ Making our life convenient
¾ Depending so much on computers may give fatal results

Question # 8 of 10 ( Start time: 07:52:26 PM ) Total Marks: 1


The gulf of execution refers to _______________ .

¾ The user’s difficulty in understanding the feedback from the system


¾ The system’s difficulty in presenting information in the output language
¾ The system’s difficulty in converting an input expression into
the correct system state transition
¾ The user’s difficulty in formulating and articulating an intention to the
system

Question # 10 of 10 ( Start time: 07:55:10 PM ) Total Marks:


1 Which of the following is not true?

¾ Utility refers to the functionality of a system


¾ Usability is concerned with adding complexity to the system
¾ Usability is concerned with making systems easy to use
¾ Poorly designed computer system can be extremely annoying to users

Question # 1 of 10 ( Start time: 09:41:29 PM ) Total Marks: 1


The command line interface is used because

¾ It is easy to understand
¾ It is demanded by DOS
¾ It is offered by Unix systems
¾ It is powerful and flexible

Question # 2 of 10 ( Start time: 09:42:00 PM ) Total Marks: 1


Which of the given statements correctly defines effectiveness in terms of one of
the usability goals?

¾ It is a very general goal and refers to how good a system at doing what it
is suppose to do.
¾ It refers to the way a system supports users in carrying out their tasks.
¾ It involves protecting the users from dangerous conditions
¾ It involves protecting the users from undesired situations

Question # 4 of 10 ( Start time: 09:43:38 PM ) Total Marks: 1


Which of the following is concerned primarily with understanding human

Come and Join Us at www.vustudents.ning.com


behavior and the mental processes that underlie it?

¾ Psychology
¾ Sociology
¾ Statistics
¾ Computer Science

Question # 5 of 10 ( Start time: 09:44:09 PM ) Total Marks: 1


HCI deals with

¾ Design of interactive system only


¾ Evaluation of interactive system only
¾ Implementation of interactive system only
¾ All of the given choices

Question # 7 of 10 ( Start time: 09:45:02 PM ) Total Marks: 1


Human beings interact with outside world, using their

¾ input channels
¾ output channels
¾ sense of sight
¾ All of the given

Question # 9 of 10 ( Start time: 09:45:56 PM ) Total Marks: 1 Visually


impaired persons can interact with outside world using their

¾ sense of sight sense of hearing


¾ both sense of touch and sense of hearing
¾ sense of touch

Question # 10 of 10 ( Start time: 09:46:18 PM ) Total Marks: 1


Which of the given statements correctly defines efficiency in terms of one of
the usability goals?

¾ It is a very general goal and refers to how good a system at doing what it
is suppose to do.
¾ It refers to the way a system supports users in carrying out their tasks.
¾ It involves protecting the users from dangerous conditions
¾ It involves protecting the users from undesired situations

Question No: 7 ( M a r k s: 1 )
__________ plays a role to bridge up the gape between the interfaces of
machines and human understanding.
► Human
► Computer

Come and Join Us at www.vustudents.ning.com


► Human Computer Interaction
► None of these

Question No: 8 ( M a r k s: 1 )
The persona’s needs are sufficiently unique to require a distinct
interface form and behavior is _____________ type of Persona.

► Primary

► Secondary

► Supplement

► Negative

Question No: 9 ( M a r k s: 1 ) http://vuzs.net


A _________is usually a collection of icons those are reminiscent of the
purpose of the various modes.

► Button

► Pointer

► Palette

► Title bar

Question No: 10 ( M a r k s: 1 )
The persona is not an actual user of the product, but is indirectly affected
by it and its use refers to _______ persona

► Primary

► Secondary

► Served

► Negative

Question No: 11 ( M a r k s: 1 )
During _____________ phase, usage and workflow patterns discovered
through ______________.

Come and Join Us at www.vustudents.ning.com


► Modeling, analysis

► Analysis, modeling

► Testing, modeling

► Testing, Analysis

Question No: 12 ( M a r k s: 1 )
The Usability Engineering lifecycle was proposed by _________.

► Deborah Mayhew
► Webster
► Barry Boehm.

► Hartson.

Question No: 13 ( M a r k s: 1 ) http://vuzs.net


Waterfall model is basically a _________ model in which each step must
be completed before the next step can be started

► Incremental

► Linear

► Iterative

► Analytical

Question # 1 of 10 ( Start time: 04:26:12 PM ) Total Marks: 1


_______ and _________ are the same things

Excise and Navigation


Excise and Testing
Excise and Evaluation
All of the given
Quiz Start Time: 04:26 PM Time Left 89
sec(s)

Question # 2 of 10 ( Start time: 04:27:25 PM ) Total Marks: 1


Scenario content and context are derived from information gathered during the
____________ phase and analyzed during the ______________ phase
(respectively)

Come and Join Us at www.vustudents.ning.com


¾ Research, modeling
¾ Modeling, implementation
¾ Research, implementation
¾ Modeling,Research

Question # 3 of 10 ( Start time: 04:28:21 PM ) Total Marks: 1


Usability testing works for _____________.

¾ Software products
¾ Hardware products
¾ All products
None of the given

Question # 4 of 10 ( Start time: 04:29:09 PM ) Total Marks: 1


What do you enjoy most about your job (or lifestyle) is an example of _________.

¾ Avoidance
¾ Motivation
¾ Exceptions
¾ Attitude-oriented questions

Question # 5 of 10 ( Start time: 04:29:46 PM ) Total Marks: 1


________ represents the Early-phase of ethnographic interviews.

¾ Clarify user roles and behaviors.


¾ Confirm patterns of use.
¾ Clarifying questions.
¾ Focused on domain knowledge.

Question # 7 of 10 ( Start time: 04:31:56 PM ) Total Marks: 1


There can only be one _________ persona per interface for a product

¾ Primary
¾ Secondary
¾ Supplemental
¾ Customer

Question # 8 of 10 ( Start time: 04:32:40 PM ) Total Marks: 1


____________ capture the non-verbal dialogue between artifact and user over
time.

¾ Persona
¾ Scenario
¾ Dialogue

Come and Join Us at www.vustudents.ning.com


¾ Design model

Question # 1 of 10 ( Start time: 04:35:42 PM ) Total Marks: 1


Number of keystrokes is the type of ____________ work.

Logical
Mnemonic
Physical
Structural

Question # 2 of 10 ( Start time: 04:36:46 PM ) Total Marks: 1


What are the most common things you do with the product is a type of ________.

¾ Goal-oriented question.
¾ System-oriented question.
¾ Workflow-oriented question.
¾ Attitude-oriented question.

Question # 3 of 10 ( Start time: 04:37:24 PM ) Total Marks: 1


Desktop applications fit into _____________ categories of posture.

¾ Two
¾ Four
¾ Five
¾ Three

Question # 5 of 10 ( Start time: 04:38:32 PM ) Total Marks: 1 Goal-oriented


context scenarios are _____________ task-oriented than key path scenario

¾ Less
¾ Alike
¾ More
¾ None of the given

Question # 6 of 10 ( Start time: 04:39:55 PM ) Total Marks: 1


____________ patterns can be applied at the conceptual level.

¾ Postural
¾ Structural
¾ Behavioral
¾ Mnemonic

Question # 7 of 10 ( Start time: 04:40:14 PM ) Total Marks: 1

Come and Join Us at www.vustudents.ning.com


____________ represent the user’s expectations of the tangible outcomes of using
a specific product.

¾ Non-user goals
¾ End goals
¾ Experience goals
¾ Life goals

Question # 8 of 10 ( Start time: 04:40:47 PM ) Total Marks: 1


Usability testing works for _____________.

¾ Software products
¾ Hardware products
¾ All products

Question # 10 of 10 ( Start time: 04:41:49 PM ) Total Marks: 1


If the user requires access via a navigational portal relatively infrequently, the
appropriate posture is ______________.

¾ Sovereign
¾ Transient
¾ Temporary
¾ None of the given

Question # 4 of 10 ( Start time: 05:13:09 PM ) Total Marks: 1


The _____________ phase sets the stage for the core of the design effort

¾ Requirement definition
¾ Modeling
¾ Implementation
¾ None of the given

Question # 5 of 10 ( Start time: 05:14:12 PM ) Total Marks: 1


If the product is new then _____________ time is usually invested in
market research.

¾ Less
¾ Less
¾ More
¾ None of the given

Question # 8 of 10 ( Start time: 05:17:50 PM ) Total Marks: 1


Which of the following is least likely to be revealed by a paper prototype?

Come and Join Us at www.vustudents.ning.com


¾ Your users don’t know the term algorithm
¾ Toolbar buttons are too small to press.
¾ The Help menu isn’t in the right place.
¾ None of the given

Question # 9 of 10 ( Start time: 05:19:06 PM ) Total Marks: 1


_______ and _________ are the same things

¾ Excise and Navigation


¾ Excise and Testing
¾ Excise and Evaluation
¾ All of the given

Question # 10 of 10 ( Start time: 05:20:31 PM ) Total Marks: 1


_____________ is needed to check that users can use the product and
like it.

¾ Coding
¾ Evaluation
¾ Guideline
¾ None of the given

Question No: 9 ( Marks: 1 ) - Please choose one


What is the fifth step in defining the requirements with respect to persona-
based design?

► Brainstorming.
► Identifying persona expectations.
► Constructing the context scenario.
► Identifying needs
Question No: 12 ( Marks: 1 ) - Please choose one
What is the main strength of the Problem Space Framework as a model of
human problem solving?

► It operates within the constraints of the human processing system


► It explains what is involved in insight
► It allows ill-defined problems to be solved
► None of these

Question No: 14 ( Marks: 1 ) - Please choose one


Over a short period of time, we find it easier to remember the string of numbers
"404 894 6743" because

► Numbers are easier to remember than arbitrary characters.

Come and Join Us at www.vustudents.ning.com


► The grouping of the numbers is significant
► Ten numbers is not that many to have to remember from working memory.
► None of these

Question No: 18 ( Marks: 1 ) - Please choose one


The established standard suite of _________ for most applications consists of the
Save As dialog, the Save Changes dialog, and the Open File dialog.

► OS management

► File management

► Data management

► Application management

Question No: 20 ( Marks: 1 ) - Please choose one


Explanatory undo is, generally, a much more pleasant feature than ___________.

► Single Undo
► Multiple undo
► Blind undo
► Incremental Undo

Question No: 21 ( Marks: 1 ) - Please choose one


Browser Title always contains the word ‘_______’.

► Home
► Default
► Error
► Browser

Question No: 22 ( Marks: 1 ) - Please choose one


Interpretation inquiry, according to Beyer and Holtzblatt, is based on a master-
apprentice model of learning.

► True
► False

Question No: 25 ( Marks: 1 ) - Please choose one


____________ are the street signs of the web.

Come and Join Us at www.vustudents.ning.com


► Site IDs
► Home pages
► Page Names
► Sections

Question No: 26 ( Marks: 1 ) - Please choose one


______________ is not like other pages; it has different burdens to bear, different
promises to keep.

► Homepage
► Form
► Navigation bar
► Sections

Question No: 27 ( Marks: 1 ) - Please choose one


The ___________ suggests identifying goals and questions first before selecting
techniques for the study
► RVM model
► DECIDE framework
► Usability testing
► Field study

Question No: 29 ( Marks: 1 ) - Please choose one


Recalling password is ____________ work.
► Logical
► Mnemonic
► Physical
► Structural

Question No: 30 ( Marks: 1 ) - Please choose one


The persona is not an actual user of the product, but is indirectly affected by it and
its use refers to _______ persona

► Primary
► Secondary
► Served
► Negative

Question ( Marks: 1 ) - Please choose one


To make an appointment you need to see a calendar and possibly
contacts you might incorporate all these together, due to____ .

Come and Join Us at www.vustudents.ning.com


Form factor
Input method
View
None of the given

Question ( Marks: 1 ) - Please choose one


What is the fourth step in defining the requirements with respect to
persona-based design?

Creating problem and vision statement.


Brainstorming.
Identifying persona expectations.
Constructing the context scenario.

Question ( Marks: 1 ) - Please choose one


Navigation represents excise except in _________.

Web
Games
Windows
Menus

Question ( Marks: 1 ) - Please choose one


If the user requires access via a navigational portal relatively
infrequently, the appropriate posture is ____________.

Sovereign
Transient
Temporary
None of the given

Question ( Marks: 1 ) - Please choose one


Because the user's interaction with a _______ program dominates his
session at the computer, the program shouldn't be afraid to take as
much screen real estate as possible.

Transient
Auxiliary
Sovereign
Daemonic

Question ( Marks: 1 ) - Please choose one


Programs that are best used full-screen, monopolizing the user’s attention
for long periods of time, are ___________ posture application.

Come and Join Us at www.vustudents.ning.com


Transient
Sovereign
Auxiliary
None of the given

Question ( Marks: 1 ) - Please choose one


Evaluations done during design to check that product continues to meet
users’ needs are known as ___________ evaluation.

Formative
Summative
Relative
None of the given

Question ( Marks: 1 ) - Please choose one


What is the second step in defining the requirements with respect
to persona-based design?

Brainstorming
Creating problem and vision statement.
Identifying persona expectations.
Constructing the context scenario.

Question ( Marks: 1 ) - Please choose one


__________ patterns can be applied at the conceptual level.

Postural
Structural
Behavioral
Mnemonic

Question ( Marks: 1 ) - Please choose one


Overviews serve a similar purpose to _______ in an interface.

Navigation
Excise
Signposts
None of the given

Question ( Marks: 1 ) - Please choose one


__________personas address the needs of customers, not end users.

Come and Join Us at www.vustudents.ning.com


Served
Negative
Customer
Supplemental

Question ( Marks: 1 ) - Please choose one


Navigation represents excise except in _________.

Web
Games
Windows
Menus

Question ( Marks: 1 ) - Please choose one


Desktop applications fit into ___________ categories of posture.
Two
Four
Five
Three

Question ( Marks: 1 ) - Please choose one


Scenario content and context are derived from information gathered
during the __________ phase and analyzed during the____________
phase (respectively)

Research, modeling
Modeling, implementation
Research, implementation
Modeling, Research

Question ( Marks: 1 ) - Please choose one


Evaluations done during design to check that product continues to meet
users’ needs are known as ___________ evaluation.

Formative
Summative
Relative
None of the given

Question ( Marks: 1 ) - Please choose one


__________ represent the user’s expectations of the tangible outcomes of
using a specific product.

Non-user goals

Come and Join Us at www.vustudents.ning.com


End goals
Experience goals
Life goals
Cs408 MCQs for FINAL TERM
by Libriansmine

► Usability testing works for _____________.

Select correct option:

Software products

Hardware products

All products None

of the given

► Transactional sites that are used for a significant part of an employee’s job should be
considered full _____________ applications.

Select correct option:

Sovereign

Transient

Temporary

None of the given

► “What site is this”, Statement represents___________.

Select correct option:

Folder

Site ID
Scroll bar

None of the given


► ____________ patterns can be applied at the conceptual level.

Select correct option:

Postural

Structural

Behavioral

Mnemonic

► __________ is a usability test in which a member of the test team sits in the room with
the participant and actively probes the participant's understanding of whatever is being
tested.

Select correct option:

Co discovery

Active intervention

Trunk test

None of the given

► Recalling password is ____________ work.

Select correct option:

Logical

Mnemonic

Physical

Structural

► _____________ is needed to check that users can use the product and like it.

Select correct option:

Coding

Evaluation

Guideline
None of the given

► Evaluations done during design to check that product continues to meet users’ needs are
known as _____________ evaluation.

► __________ is the extra work that satisfies the needs to achieve our objective.

Select correct option:

Evaluation

Excise

Testing

None of the given

► In DECIDE frame work _______________ comes just after choosing the evaluation
paradigm and techniques.

► Number of keystrokes is the type of ____________ work.

Select correct option:

Logical

Mnemonic

Physical
Structural

► _________ has a goal of assessing whether the product works according to its
specifications.

Select correct option:

Trunk Test

Quality assurance

Both above

None of the above

► In usability test, which is a technique from given below opttions, in which you have two
participants work together to perform the tasks.

Select correct option:

Active Intervention

Co-Discovery

Trunk test

None of the given

► Information sites with daily-updated information will naturally attract ____________


users more than a monthly-updated site.

Select correct option:

Repeat

Infrequent

Nonuser

None of the given

► If the user requires access via a navigational portal relatively infrequently, the appropriate
posture is ______________.

Select correct option:

Sovereign

Transient
Temporary

None of the given

► Because the user's interaction with a _________ program dominates his session at the
computer, the program shouldn't be afraid to take as much screen real estate as possible.

Select correct option:

Transient

Auxiliary

Sovereign

Daemonic

► ____________ is particularly useful early in design. It is excellent technique to use with


the prototype, because it provides a wealth of diagnostic information.

Select correct option:

Co-discovery

Active intervention

Splendid research

None of the given

► To make an appointment you need to see a calendar and possibly contacts you might
incorporate all these together, due to ______ .

Select correct option:

Form factor

Input method

View

None of the given

► Global Navigation system is also called__________.

Select correct option:

Site ID
Persistent Navigation

Both

None

► Engineering a user interface involves a quantitative engineering style of working in


which measurements are used to judge the quality of interface. Hence ____________ is
appropriate.

Select correct option:

Usability testing

Field study

Predictive evaluation

DECIDE framework

► ______________ applications are great platforms for creating an environment rich in


visual feedback for the user.

Select correct option:

Sovereign

Transient

Auxiliary

Daemonic

► Overviews serve a similar purpose to _________ in an interface.

Select correct option:

Navigation

Excise

Signposts

None of the given

► Panes, frames and other containers on screen is a kind of _____________

Select correct option:


Functional and data elements

View

Input method

None of the given

► _____________ are perhaps the least-documented patterns, but they are nonetheless in
widespread use.

► _____________ is needed to check that users can use the product and like it.

Select correct option:

Coding

Evaluation

Guideline

None of the given

► This is not a characteristic of usability test

► Desktop applications fit into _____________ categories of posture.

Select correct option:


Two

Four

Five

Three

► Navigation represents excise except in ___________.

Select correct option:

Web

Games

Windows

Menus

► _________is an early release of a product to a few users.

Select correct option:

Surveys

Beta test

Focus

None of the above

► _____________ is like the building name for a website.

Site ID

Navigation

Section

None of the given

► ___________ is the process of selecting things to concentrate on, at a point in time, from
the range of possibilities available.

Perception and recognition

Attention
Learning

► name of the document should be shown on the application’s__________.

A. Menu bar

B. Title bar

C. Title bar and menu bar

D. Not Title bar and not menu bar

► Exploring how children talk together in order to see if an innovative new groupware
product would help them to be more engaged would probably be better informed by a
______________.

Usability testing

Field study

Predictive evaluation

DECIDE framework

► _________ is the remarkable facility that lets us reverse a previous action.

Redo

Undo

Repeat

Delete

► ____________ are the street signs of the web.

Site IDs

Home pages

Page Names

Sections

► ______________ is not like other pages; it has different burdens to bear, different promises
to keep.
Homepage

Form

Navigation bar

Sections

► ___________ suggests identifying goals and questions first before selecting techniques for
the study

► Conventional wisdom says that ________ tell the user when he has made some mistake.

Program crash

System stuck

Error messages

Metadata

► People frequently counter the idea of ________feedback with arguments that users don’t
like it

► When developing ____________, plan to keep them short, straightforward and avoid asking
too many

Videos
Documentation
Interview questions
Contextual enquiry

► HCI deals with---------------------------


Design of interactive system only
Evaluation of interactive system only
Implementation of interactive system only
All of the given choices

____________ frequent your primary personas access the site, _____________ transient a
stance the site needs to take (respectively).

More, Less
Less, more
Less, less
None of the given

► When you engaged in a conversation you are more attentive what the other person is
saying.It is called?

Focused Attention
Voluntary Attention
Involuntary attention
Divided Attention

► Drive a vehicle while holding a conversation with a passenger is the example of _________ .

Focused Attention

Voluntary Attention

Involuntary attention

Divided Attention

► ____________ minimizes errors.

Affordance

Constraints

Visibility
Affordance and Visibility

► ___________ are dragged down from the title at the top of the screen.

Pull Down Menus

Main Menus

Icons

Buttons

► ________ aspect (s) gives us hints and tips about using and creating user interface idioms,
like dialog boxes and pushbuttons.

► Efficient.
► Strategic.
► Tactical.
► Reliable.

The Usability Engineering lifecycle was proposed by _________.


Harston
Webster Barry
Boehm Deborah
Mayhew

► Ali is looking at how interface designers went about their work. He identified two different
modes of activity: one is _________ and other is ___________ .
Analytic mode, synthetic mode
Evaluation mode, testing mode
Analyze mode, feedback mode
Implementation mode, task mode

► Scenario content and context are derived from information gathered during the
____________ phase and analyzed during the ______________ phase (respectively)

Modeling, Implementation
Modeling, Research
Research, Implementation
Research, Modeling

► Alia is used to describe design problem /solution by ---------------that is an imagined or


projected sequence of events, especially any of several detailed plans or possibilities
Persona
Scenario
Flowchart
Brainstorming

52. The _____________ is a concrete expression of the more abstract structure of the site.

Skeleton
Scope
Strategy
Structure

► Ali wants to develop an application that incorporates print, audio video media and software
applications on a single place. Which of the following will help in this regard?

MS World
Worldwide Web
Television Newspaper

► XYZ Ltd is well reputed software house; they make a significant investment in building ------
------------- that encourages greater customer loyalty.

Visual Symbols
Grouping
Brand Equity
Harmony

► Around 63% of software projects exceed their cost estimates. The top four reasons for

This are:

A– Frequent requests for changes from users


B– Overlooked tasks
C– Users' lack of understanding of their own requirements
D– Insufficient user-analyst communication and understanding

Only A
Only B
Only D
ABC&D

► ________of the potential sales from a site are lost because people cannot find the item they
are looking for
50%

57. Human eye is very sensitive to ______

Air
Smoke
Light
Sand

58 . You must understand the constraints of _________

Technology

59. Top of the web page contain the _________

Menu
Title bar
Home icon
Back button

► If the user wants to _________ place the document somewhere in the file system

hierarchy, he can request this function from the menu.

Explicitly
Implicitly
Habitually
Properly

61. Which of the following is least likely to be revealed by a paper prototype?

Your users don’t know the term algorithm


Toolbar buttons are too small to press.
The Help menu isn’t in the right place.
None of the given

► is the least technical way of collecting data, but it can be difficult and tiring to write and
observe at the same time.
Audio recording.
Taking notes.
Observation
Video

► Every page within a site should contain a brief _________ that accurately describes the
content of the page.

Site ID.
Header
Local Navigation.
Search button

64. What is the fifth step in defining the requirements with respect to persona-based design?

Brainstorming.
Identifying persona expectations.
Constructing the context scenario
Identifying needs

► You can load a VCR tape the right way because of _____________.
Physical constraints
Logical constraints
Cultural constraints
None of these

66.____________ are unintentional while _____________ occur through conscious deliberation.

Slips, mistakes(page:100)
Errors, slips
Mistakes, errors
Mistakes, slips

► What is the main strength of the Problem Space Framework as a model of human problem
solving?

It operates within the constraints of the human processing system


It explains what is involved in insight
It allows ill-defined problems to be solved
None of these

► Over a short period of time, we find it easier to remember the string of numbers "404 894
6743" because

Numbers are easier to remember than arbitrary characters.


The grouping of the numbers is significant
Ten numbers is not that many to have to remember from working memory.
None of these

► environments are environments that are user and context aware.

Attentive
Non-attentive
Visual
Sensing

► involves watching and listening to users.

Observation
Evaluation
Qualitative research
Interaction

► The established standard suite of _________ for most applications consists of the Save As
dialog, the Save Changes dialog, and the Open File dialog.

OS management
File management
Data management
Application management

72. Explanatory undo is, generally, a much more pleasant feature than ___________.

Single Undo
Multiple undo
Blind undo
Incremental Undo
► Browser Title always contains the word ‘_______’.

Home
Default
Error
Browser

► Interpretation inquiry, according to Beyer and Holtzblatt, is based on a master-apprentice model of learning.

True
False

► Roughly _____________ percent of the male population has some degree of color blindness.

10
20
30
40

► The persona is not an actual user of the product, but is indirectly affected by it and its use refers to _______ persona

Primary
Secondary
Served
Negative
i

► Solved by libriansmine
MAR
21,2015
Solved MCQS
From Midterm and also from daily based
quizzes(1 to 22 lectures)
BS110401050 BS110401050@vu.edu.pk Arslan.arshad01@gmail.com AKMP01

Question # 1: Which of the following is proportional to the amplitude of the sound; the frequency
remains constant?

o Pitch

o Loudness page # 70

o Timber

o None of these

Question # 2: The human eye has approximately rods and cones per eye

o 120 million , 6 million Page # 56

o 190 million , 3 billion

o 150 billion , 6 million

o None of the given

The human eye has approximately 120 million rods 6 million cones per eye
Question # 3: What are the three types of reasoning?

o Deductive, Productive and Inductive

o Abdicative, Inductive and Deductive Page # 89

o Inductive, Abdicative and Reproductive

o Productive, Reproductive and Deductive

Reasoning
Reasoning is the process by which we use the knowledge we have to draw conclusions or infer something new about
the domain of interest. There are a number of different types of reasoning:
• Deductive reasoning
• Inductive reasoning
• Abdicative reasoning

Question # 4: Which of the following is not one of the goals of HCI?

o To produce usable systems

o To produce safe systems

o To produce non-functional systems Page # 30

o To improve effectiveness of the systems

The goals of HCI are to produce usable and safe systems, as well as functional systems. These goals can be summarized
as „to develop or improve the safety, utility, effectiveness, efficiency and usability of systems
Question # 5: Which of the following is considered as the most complex species?

o Humans Page # 18

o Computers

o Animals

o Birds
Human beings are the most interesting and fascinating specie on planet. They are the most complex living being on the earth

Question # 6: Totality of characteristics of an entity that bear on its ability to satisfy stated and implied needs
are called .

o Quality Page # 40 also visit this link click here for more detail

o Effectiveness

o Standard

o Quantity

Quality is essential in all we do and particularly in our chosen field of work.


Quality has been defined by the International Standards Organization (ISO) as:
The totality of characteristics of an entity that bear on its ability to satisfy stated or implied needs.

Question # 7: Which of the following is not true regarding "cones"?

o A type of receptor in eye

o more sensitive to light Page # 56

o different types of cones are sensitive to different wavelengths of light

o eye has approximately 6 million cones

Cones are the second type of receptor in the eye. They are less sensitive to light than the rods and can therefore tolerate
more light. There are three types of cone, each sensitive to a different wavelength of light. This allows color vision. The
eye has approximately 6 million cones, mainly concentrated on the fovea.
Question # 8: Which of the following is a true statement?

o Human-computer interface specialists are user-centered and software engineers are


system centered Page # 21

o Human-computer interface specialists are system-centered and software engineers are user-
centered

o Human-computer interface specialists and software engineers, both are system-centered.

o Human-computer interface specialists and software engineers, both are user-centered.

Software Engineering and HCI


There is a basic fundamental difference between the approaches taken by software engineers and human-computer
interaction specialists. Human-computer interface specialists are user-centered and software engineers are system-
centered.

Question # 9: Which of the following is not true?

o Utility refers to the functionality of a system

o Usability is concerned with adding complexity to the system page # 30

o Usability is concerned with making systems easy to use

o Poorly designed computer system can be extremely annoying to users

Utility refers to the functionality of a system or, in other words, the things it can do. Improving effectiveness and
efficiency are self-evident and ubiquitous objectives. The promotion of safety in relation to computer systems is of
paramount importance in the design of safety-critical systems. Usability, a key concept in HCI, is concerned with
making systems easy to learn and easy to use. Poorly designed computer system can be extremely annoying to users, as
you can understand from above described incidents.
Question # 10: Formal methods are used to represent

o Architecture aspects of software systems only

o Procedural aspects of software systems only

o Both Architecture and procedural aspects of software systems Page # 21

o None of the given

Formal methods have been developed to represent data, architectural, and procedural aspects of a software system

Question # 11: Which interface system is based on the question / answer dialogue?

o Command Line Interfaces

o Query interfaces Page #


127 and 128
o Menus

o Natural Language Interfaces

Question # 12: Choice of operations and services are offered on the screen through

o Pointers

o Toolbars

o Menus Page # 131

o None of the given

A menu presents a choice of operations or services that can be performed by the system at a given time.
Question # 13: What will be the gulf of execution if the user is able to formulate and perform the actions easily;

o Smaller Page # 122

o Greater

o Balanced

o None of the Given

Question # 14: Form-filling interfaces are used for

o data entry Page # 128

o data integration

o data manipulation

o data definition

Form-filling interfaces are used primarily for data entry but can be useful in data retrieval applications.

Question # 15: Which of the following is flaw in waterfall model?

o requirements change overtime


Page # 150
o maintenance can not be possible at the end

o testing should be done before requirements gathering

o All at the given

Flaws of waterfall model


One of the main flaws with this approach is that requirements change over time.
Question # 16: There are two aspects in which we can understand the nature of usability A. Strategic and
Tactical. B. Efficiency and Strategy C. Tactical and Reliability D. Efficiency and Reliability

o Only A Page #
143
o Only C

o A and D

o B and C

we can understand The nature of usability in two aspects


• Strategic
• Tactical

Question # 17: Once the user input into system, the input language is translated into the language as operations to
be performed by the system.

o Task

o Core Page # 123

o Both

o None of the above

The only way the user can manipulate the machine is through the input, and so the task must be articulated within the
input language, the input language is translated into the core language as operations to be performed by the system.
Question # 18: User and system has its own unique language the language used by the system is called .

o Core Page # 123

o Task

o Both

o None of the above

The system‟s language is referred as the core language and the user‟s language is referred as the task language

Question # 19: Which of the following is used to toggle between two states?

o check boxes

o Radio buttons
Page # 133
o Toolbars

o menus

Radio Buttons
Buttons can also be used to toggle between two states

Question # 20: Which of the following is a text entry device?

o Keyboard Page # 110

o mouse

o monitor

o touch pad

Text entry devices: There are many text entry devices as given below:
Keyboard The most common method of entering information into the computer is through a
Keyboard QWERTY keyboard, Alphabetic keyboard, Dvorak Keyboard, Chord keyboards , Phone pad and T9
entry
Question # 21: "Mistakes" and "Slips" are two forms of

o Errors Page # 100

o goals

o evaluation

o objectives

There are various types of errors. Norman has categorized them into two main types, slips and mistakes:

Question # 22: The model should match the model.

o User, conceptual

o Conceptual, mental Page # 94 Click here for more Detail

o Mental, central

o Conceptual, central

Question # 23: Which of the following is/are aspect(s) of human action?

o execution and evaluation Page # 99

o evaluation and efficiency

o efficiency and execution

o efficiency

Human action has two aspects, execution and evaluation.


Execution: involves doing something.
Evaluation: is the comparison of what happened in the world with what we wanted to happen
Question # 24: Building things from user's perspective is called

o Functionality

o Usability page # 31

o Portability

o None of the given

Usability goals
To recap, usability in generally regarded as ensuring that interactive products are easy to learn, effective to use, and
enjoyable from user perspective.

Question # 25: Certain factors that help us to interact with graphical based systems

o Indirect manipulation

o Direct manipulation
Page # 86
o Both

o None of the given

GUI and direct manipulation interface are good environments for supporting this kind of learning

Question # 26: If you are taking lecture and suddenly you hear music or voices from the other room. It is
called?

o Focused attention

o Divided attention

o Voluntary attention

o Involuntary attention Page # 70

An everyday example of an involuntary act is being distracted from working when we can hear music or voices in the
next room
Question # 27: Which of the following is a long-term individual difference?

o Age

o Sex Click here for more Detail

o Fatigue

o Color

Individual Differences Long term : Sex , Physical abilities , Intellectual abilities

Question # 28: refers to how good a system at doing What it is supposed to do?

o Safety

o Usability

o Efficiency

o Effectiveness Page # 31

Effectiveness
It is a very general goal and refers to how good a system at doing what it is suppose to do.

Question # 29: As a software designer, which aspects guide you to think about the Way in which the user and the
idiom interact?

o Tactical

o Manual

o Technical

o StrategicPage # 143
Strategic aspect guides us to think about user interface idioms – in other words, the way in which the user and the idiom
interact.
Question # 30: Adeel renders financial services in “ABC” bank .He specialized in his field. What he is called as
from the following?

o Society of manufacturing engineer (SME)

o Subject matter expert (SME) Page


# 168
o Small and medium enterprise (SME)

o Subject master engineering (SME)

Subject matter expert (SME)

Question # 31: Scenario content and context are derived from information gathered during
the phase and analyzed during the phase (respectively)

o Modeling, implementation

o Modeling, Research

o Research, implementation

o Research, modeling
Page # 100
Scenario content and context are derived from information gathered during the Research phase and analyzed during the
modeling phase.

Question # 32: What is a semantic network?

o A model of long-term memory Page # 82

o A record of our memory of events

o The part of the brain which allows us to remember things

o A mechanism for improving memory


Long-term memory structure
There are two types of long-term memory: episodic memory and semantic memory.
Question # 33: Which of the following is least likely to be revealed by a paper prototype?

o Your users don't know the term algorithm

o Toolbar buttons are too small to press Click here for more Detail

o The Help menu isn't in the right place

o Radio buttons are too small

Question # 34: is a powerful, multipurpose design tool that helps overcome several problems
that currently plague the development of digital products.

o Scenario

o Persona Page # 187

o Prototype

o None

The persona is a powerful, multipurpose design tool that helps overcome several problems that currently plague the
development of digital products.

Question # 35: The difference between the intentions and allowable actions is the:

o Gulf of Execution Page # 103

o Gulf of Evaluation

o Both of the above options

o None of these

The difference between the intentions and allowable actions is the gulf of execution.
Question # 36: Attention is the process of selecting things to concentrate on, at a point in time, from the range
of .

o Possibilities available Page # 76

o Time Available

o None of these

Attention is the process of selecting things to concentrate on, at a point in time, from the range of possibilities available.

Question # 37: The goals of HCI are:

o Usability and User Experience Page # 30

o Learn ability and Comfort

o Tasks and Goals

o All of the above options

Question # 38: WYSIWYG stands for .

o Where you see is where you get

o What you see is what you get Page # 36

o When you see it when you get

WYSIWYG (what you see is what you get)


Question # 39: is proportional to the amplitude of the sound.

o Pitch

o Loudness Page # 70

o Timber

o None of the given

Loudness is proportional to the amplitude of the sound; the frequency remains constant

Question # 40: Which of the given statements correctly defines effectiveness in terms of one of the usability
goals?

o It is a very general goal and refers to how good a system at doing what it is suppose to do.
Page # 31

o It refers to the way a system supports users in carrying out their tasks.

o It involves protecting the users from dangerous conditions

o It involves protecting the users from undesired situations

Effectiveness: It is a very general goal and refers to how good a system at doing what it is suppose to do.

Question # 41: Which of the given statements correctly defines efficiency in terms of one of the usability goals?

o It is a very general goal and refers to how good a system at doing what it is suppose to do.

o It refers to the way a system supports users in carrying out their tasks. Page # 31

o It involves protecting the users from dangerous conditions

o It involves protecting the users from undesired situations

Efficiency : It refers to the way a system supports users in carrying out their tasks.
Question # 42: User personas that are not primary or secondary are personas.

o Served

o Negative

o Customer

o Supplemental Page # 196

Supplemental personas
User personas that are not primary or secondary are supplemental personas

Question # 43: Which of the following is the comparison of what happened in the world with what we wanted to
happen?

o Action

o Evaluation Page
# 99
o Execution

o None of these

Action Cycle: Human action has two aspects, execution and evaluation. Execution involves doing something. Evaluation is
the comparison of what happened in the world with what we wanted to happen.

Question # 44: Research can tell you about what, how, many and why in rich,
multivariate detail.

o Quantitative

o Qualitative Page # 166

o SME

o None of these
Qualitative research can tell you about what, how and why in rich, multivariate detail.
Question # 45: Requirements specification involves:

A. Capturing the functional requirements of the system architecture


B. Eliciting information about the Work domain from the customer.

o Only A

o Only B Click here for more Detail

o Both A and B

o Not A and Not B

Question # 46: What are the Most common things you do with the product is a type of _ .

o Goal- oriented question

o System - oriented question Page # 182

o Workflow- oriented question

o Attitude-oriented question

System-oriented questions
Function
What are the most common things you do with the product?

Question # 47: What do you procrastinate on? is type of

o Priorities

o Avoidance
Page # 183
o Exceptions

o Preference

Avoidance
What would you prefer not to do?
What do you procrastinate on?
Question # 48: Cognitive process involves encoding and recalling knowledge and acting
appropriately

o Attention

o Reduction

o Memory Click here for more Detail and also Read PPT slides

o Encoding

Memory : Involves encoding and recalling knowledge and acting appropriately

Question # 49: During usage and workflow patterns discovered through analysis of the field
research and interviews are synthesized into domain and user models

o Research

o Modeling Page # 159

o Framework

o Interview

Modeling: During the modeling phase, usage and workflow patterns discovered through analysis of the field research
and interviews are synthesized into domain and user models.

Question # 50: Transactional sites that are used for a significant part of an employee‟s job should be considered
full applications.

o Sovereign
Page # 30
o Transient

o Temporary

o None of the given

Transactional sites that are used for a significant part of an employee's job should be considered full sovereign
applications.
Question # 51: is the process of selecting things to concentrate on, at a point in time,
from the range of possibilities available.

o Perception and recognition

o Attention Page # 76

o Learning

o None of these
Attention is the process of selecting things to concentrate on, at a point in time, from the range of possibilities available.

Question # 52: HCI deals with .

o Design of interactive system only

o Evaluation of interactive system only

o Implementation of interactive system only

o All of the given choices

Question # 53: When you engaged in a conversation you are more attentive what the other person is saying. It is
called?

o Focused Attention Page


# 78
o Voluntary Attention

o Involuntary attention

o Divided Attention

Focused Attention
For example, when engaged in a conversation it is usual to attend to what the other person is saying.
Question # 54: Drive a vehicle while holding a conversation with a passenger is the example of
.

o Focused Attention

o Voluntary Attention

o Involuntary attention

o Divided Attention Page # 78

Divided Attention
As we said, we may be skilled at carrying on the conversation while intermittently observing what the person we want to talk
to is doing. When we attempt to attend to mire than one thing at a time, as in the above example, it is called divided attention.
Another example that is often used to illustrate this intentional phenomenon is being able to drive while holding a
conversation with a passenger.

Question # 55: minimizes errors.

o Affordance

o Constraints

o Visibility Page #
104
o Affordance and Visibility

Question # 56: are dragged down from the title at the top of the screen.

o Pull Down Menus


Page # 132
o Main Menus

o Icons

o Buttons

Pull-down menus are dragged down from the title at the top of the screen, by moving the mouse pointer into the title
par area and pressing the button.
Question # 57: aspect (s) gives us hints and tips about using and creating user interface idioms,
like dialog boxes and pushbuttons.

o Efficient

o Strategic

o Tactical Page # 143

o Reliable

Tactical aspects give us hints and tips about using and creating user interface idioms, like dialog boxes and pushbuttons.

Question # 58: The Usability Engineering lifecycle was proposed by .

o Hartson

o Webster

o Barry Boehm

o Deborah Mayhew
Page # 153
The Usability Engineering lifecycle was proposed by Deborah Mayhew in 1999.

Question # 59: Ali is looking at how interface designers went about their work. He identified two different modes
of activity: one is and other is _.

o Analytic mode, synthetic mode


Page # 153
o Evaluation mode, testing mode

o Analyze mode, feedback mode

o Implementation mode, task mode

Two different modes of activity: analytic mode and synthetic mode.


Question # 60: Alia is used to describe design problem /solution by that is an
imagined or projected sequence of events, especially any of several detailed plans or possibilities

o Persona

o Scenario

o Flowchart

o Brainstorming

Question # 61: are individual and isolated regions within display that can be selected by the
user to invoke specific operations.

o Buttons Page # 133

o Pointers

o Menus

o Windows

Buttons are individual and isolated regions within display that can be selected by the user to invoke specific operations.
Question # 62: Around 63% of software projects exceed their cost estimates. The top four reasons for
This are:
A– Frequent requests for changes from users B–
Overlooked tasks
C– Users' lack of understanding of their own requirements
D– Insufficient user-analyst communication and understanding

o Only A

o Only B

o Only D

o A B C & D Page #
27
Around 63% of software projects exceed their cost estimates. The top four reasons for this are:
– Frequent requests for changes from users
– Overlooked tasks
– Users' lack of understanding of their own requirements
– Insufficient user-analyst communication and understanding

Question # 63: of the potential sales from a site are lost because people cannot find the item they
are looking for

o 50% Page # 141

o 40%

o 30%

o 10%

50% of the potential sales from a site are lost because people cannot find the item they are looking for
Question # 64: Human eye is very sensitive to

o Air

o Smoke

o Light Page # 55 and 56

o Sand

Question # 65: Top of the web page contain the

o Menu

o Title bar

o Home icon

o Back button

Question # 66: You can load a VCR tape the right way because of .

o Physical constraints Page # 106

o Logical constraints

o Cultural constraints

o None of these

Physical constraints: Physical constraints refer to the way physical objects restrict the movement of things. For example,
the way a external disk can be placed into a disk drive is physically constrained by its shape and size, so that it can be
inserted in only one way. Likewise, keys on a pad can usually be pressed in only one way.
Question # 67: are unintentional while occur through conscious
deliberation.

o Slips, mistakesPage # 100

o Errors, slips

o Mistakes, errors

o Mistakes, slips

Slips
Slips are unintentional. They happen by accident, such as making typos by pressing the wrong key or selecting wrong menu
item by overshooting. The most frequent errors are slips, especially in well-learned behavior.

Mistakes
Mistakes occur through conscious deliberation. An incorrect action is taken based on an incorrect decision. For example,
trying to throw the icon of the hard disk into the wastebasket, in the desktop metaphor, as a way of removing all existing files
from the disk is a mistake. A menu option to erase the disk is appropriate action.

Question # 68: What is the main strength of the Problem Space Framework as a model of human problem solving?

o It operates within the constraints of the human processing system Page # 91

o It explains what is involved in insight

o It allows ill-defined problems to be solved

o None of these

Reference:
Click here for more detail
Question # 69: Over a short period of time, we find it easier to remember the string of numbers "404 894 6743"
because:

o Numbers are easier to remember than arbitrary characters.

o The grouping of the numbers is significant

o Ten numbers is not that many to have to remember from working memory.

o None of these

Question # 70: Interpretation inquiry, according to Beyer and Holtzblatt, is based on a master apprentice model of
learning.

o True

o False Page # 176

Contextual inquiry: Contextual inquiry, according to Beyer and Holtzblatt, is based on a master- apprentice
model of learning: observing and asking questions of the users as if she is the master craftsman and he interviews
the new apprentice.

Question # 71: The persona is not an actual user of the product, but is indirectly affected by it and its use refers to
persona

o Primary

o Secondary

o Served Page # 159

o Negative
Served: the persona is not an actual user of the product, but is indirectly affected by it and its use
Question # 72: Which of the following is not a secondary color?

o Green

o Orange

o Purple

o Blue Page # 63

Secondary Colors: These are the colors formed by mixing the primary colors.
SECONDARY COLORS
Green, orange and purple

Question # 73: Which of the following requires less cognitive effort than others?

o Listening Page # 87

o Speaking

o Hearing

o None of the given

Listening require less cognitive effort than reading or speaking.

Question # 74: We are deficient in our development , not in our development


(respectively).

o Process, Tools
Page # 23
o Tools, Process

o Tools, Methodology

o None of these

We are deficient in our development process, not in our development tools.


Question # 75: The command line interface is used because:

o It is easy to understand

o It is demanded by DOS

o It is offered by UNIX systems

o It is powerful and flexible Page # 127

Command line interface are powerful in that they offer direct access to system functionality, and can be combined to apply a
number of tools to the same data. They are also flexible: the command often has a number of options or parameters that will vary
its behavior in some way, and it can be applied to many objects at once, making it useful for repetitive tasks.

Question # 76: Which of the following is concerned primarily with understanding human behavior and the
mental processes that underlie it?

o Psychology Page # 43

o Sociology

o Statistics

o Computer Science

Psychology is concerned primarily with understanding human behavior and the mental processes that underlie it.

Question # 77: plays a role to bridge up the gape between the interfaces of machines and
human understanding.

o Human

o Computer

o Human Computer Interaction Page # 14

o None of these

HCI plays a role to bridge up the gape between the interfaces of machines and human understanding that we have
seen in the previous examples.
Question # 78: The persona‟s needs are sufficiently unique to require a distinct interface form and behavior is
type of Persona.

o Primary Page # 159

o Secondary

o Supplement

o Negative

Primary: the persona’s needs are sufficiently unique to require a distinct interface form and behavior

Question # 79: A is usually a collection of icons those are reminiscent of the purpose of the
various modes.

o Button

o Pointer

o Palette Page # 133

o Title bar

A palette is usually a collection of icons that are reminiscent of the purpose of the various modes.

Question # 80: During phase, usage and workflow patterns discovered through
.

o Modeling, analysis Page # 159

o Analysis, modeling

o Testing, modeling

o Testing, Analysis

Modeling: During the modeling phase, usage and workflow patterns discovered through analysis of the field research
and interviews are synthesized into domain and user models.
Question # 81: The Star lifecycle was proposed by .

o Deborah Mayhew

o Webster

o Barry Boehm

o Hartson Page #
152
In 1989, the Star lifecycle model was proposed by Hartson and Hix.

Question # 82: Waterfall model is basically a model in which each step must be
completed before the next step can be started.

o Incremental

o Linear Page # 149

o Iterative

o Analytical

The waterfall lifecycle was the first model generally known in software engineering and forms the basis of many lifecycle in
use today. This is basically a linear model in which each step must be completed before the next step can be started.

Question # 83: What do you enjoy most about your job (or lifestyle) is an example of .

o Avoidance

o Motivation Page # 183

o Exceptions

o Attitude-oriented questions

Motivation
What do you enjoy most about your job (or lifestyle)?
What do you always tackle first?
Question # 84: represents the Early-phase of ethnographic interviews.

o Clarify user roles and behaviors

o Confirm patterns of use

o Clarifying questions

o Focused on domain knowledge


Page # 181
Early-phase
• Exploratory
• Focused on domain knowledge
• Open-ended questions

Question # 85: There can only be one persona per interface for a product.

o Primary Page # 196

o Secondary

o Supplemental

o Customer

Primary personas represent the primary target for the design of an interface. There can be only one primary persona per interface
for a product.

Question # 86: What are the most common things you do with the product is a type of .

o Goal-oriented question.

o System-oriented question. Page #


182
o Workflow-oriented question.

o Attitude-oriented question.

System-oriented questions
Function: What are the most common things you do with the product?
Question # 87: represent the user‟s expectations of the tangible outcomes of using a specific
product.

o Non-user goals

o End goals Page # 192

o Experience goals

o Life goals

End goals represent the user‟s expectations of the tangible outcomes of using specific product.

Question # 88: personas address the needs of customers, not end users.

o Served

o Negative

o CustomerPage # 197

o Supplemental

Customer personas address the needs of customers, not end users. Typically, customer personas are treated like
secondary personas.

Question # 89: Persona is not context specific, so it can be reused easily.

o True

o False Page # 189

Personas and reuse


– Personas context-specific
• Cannot be reused across products
Slide No. 22 Page # 21/61
Question # 90: The size and shape of a compact disk, is a type of constraint.

o Physical Page #
106
o Logical

o Cultural

o None of these

Physical constraints
Physical constraints refer to the way physical objects restrict the movement of things. For example, the way a external disk
can be placed into a disk drive is physically constrained by its shape and size, so that it can be inserted in only one way.
Likewise, keys on a pad can usually be pressed in only one way.

Question # 91: Which of the following device can not be useful for a visually impaired person?

o a typical keyboard

o a typical monitor

o a typical speaker

o a typical processor

Monitors are useless to the visually impaired or blind person.

Question # 92: The gulf of execution refers to _.

o The user‟s difficulty in understanding the feedback from the system

o The system‟s difficulty in presenting information in the output language

o The system‟s difficulty in converting an input expression into the correct system state transition

o The user’s difficulty in formulating and articulating an intention to the system


Question # 93: Learnability, flexibility and robustness are three main usability principles that can be considered as
general headings for standards and guidelines generation. Which of the following are also high level usability
categories that can guide standards and guidelines generation?

(i) Effectiveness
(ii) Efficiency
(iii) Fault tolerance
(iv)Satisfaction

Select correct option:

o (i) & (ii)


Page # 31
o (i), (ii) & (iv)

o (ii) & (iii)

o (ii) & (iv)

usability is broken down into the following goals:


• Effective to use (effectiveness)
• Efficient to use (efficiency)
• Safe to use(safety)
• Have good utility (utility)
• Easy to learn (learnability)
• Easy to remember how to use (memorability)

Question # 94: Which of the following is not one of the primary colors?

o Red

o Yellow

o Blue

o Green Page # 62

Primary Colors
In traditional color theory, these are the 3 pigment colors that cannot be mixed or formed by any combination of other colors.
All other colors are derived from these 3 hues
PRIMARY COLORS
Red, yellow and blue
Question # 95: Human beings interact with outside world, using their

o input channels

o output channels

o sense of sight

o All of the given


Page # 54
A person‟s interaction with the outside world occurs through information being received and sent: input and output.
Sight may be used primarily in receiving information from the computer, but it can also be used to provide information
to the computer, for example by fixating on a particular screen point when using an eye gaze system.

Question # 96: Visually impaired persons can interact with outside world using their

o Sense of sight

o Sense of hearing

o Both sense of touch and sense of hearing

o Sense of touch

Question # 97: refers to the relationship between controls and their effects in the world.

o Visibility

o Affordance

o Mapping Page # 107

o None of the given

Mapping
This refers to the relationship between controls and their effects in the world
Question # 98: is a very general goal of Usability and refers to how good a system at doing
what it is suppose to do.

o Effectiveness Page # 31

o Efficiency

o Utility

o None of the given

Question # 99: is what goes on in out heads when we carry out our everyday
activities?

o Cognition
Page # 47
o Learnability

o Memorability

o None of the given

Question # 100: is the process by which we use the knowledge we have to draw
conclusions or infer something new about the domain of interest.

o Decision Making

o Reasoning Page # 89

o Problem Solving

o None of the given


Question # 101: research helps us understand the domain, context and constraints of a product in
different, more useful ways than research do.

o Qualitative, Quantitative Page # 167

o Quantitative, Qualitative

o Qualitative, Deductive

o None of them

Question # 102: In an organization individuals may keep their own records, or there may be local gurus. This
statement concerns with .

o Paper work and computer work

o Spatial and temporal organizations

o Organizational memory
Page # 176
o None of these

Organizational memory
Formal documents are not the only way in which things are remembered within an organization. Individuals may keep their
own records, or there maybe local gurus.

Question # 103: Using icons on the desktop to represent operations is a type of


constraint.

o Physical

o Logical

o Cultural Page # 106

o None of these
Question # 104: is a term used to refer to an attribute of an object that allows people to know
how to use it.

o Visibility

o Affordance Page
# 105
o Constraint

o None of these

Question # 105: minimizes errors.

o Affordance

o Visibility Page #
104
o Constraints

o None of these

Question # 106: Models are used in design to:

o Generate the design

o Evaluate the design

o Generate and evaluate the design Click here for more Detail

o None of the given


Question # 107: Which are the most significant senses for the average person when it comes to interacting with a
computer?

o Sight and hearing

o Hearing, touch and smell

o Hearing and touch

o Sight, hearing and touch Click here for more Detail

The major senses in human interaction are vision, hearing and touch.

Question # 108: refers to the way a system supports users in carrying out their tasks.

o Efficiency Page # 31

o Effectiveness

o Utility

o None of the given

Efficiency
It refers to the way a system supports users in carrying out their tasks.

Question # 109: are GUIs that consists of electronic counterparts to physical objects in the real
world to match the knowledge requirements of users.

o User Interaction Models

o Conceptual Models

o Interface Metaphors Page # 51

o None of the given

Interface metaphors are (these are GUIs that consists of electronic counterparts to physical objects in the real world) to
match the knowledge requirements of users.
Question # 110: language tends to be grammatical while language is often
Ungrammatical.

o Written, spoken
Page # 87
o Spoken, written

o Verbal, non-verbal

o None of the given

Written language tends to be grammatical while spoken language is often ungrammatical.

Question # 111: Aspect gives us hints and tips about using and creating user interface idioms.

o Strategic

o Tactical Page # 143

o Operational

o None of the given choices

Strategic aspect guides us to think about user interface idioms – in other words, the way in which the user and the
idiom interact. Tactical aspects give us hints and tips about using and creating user interface idioms, like dialog boxes
and pushbuttons.

Question # 112: Faisal is looking at how interface designers went about their work. He identified two different
modes of activity: one is and other is _.

o Analytic mode, synthetic mode Page # 153

o Evaluation mode, testing mode

o Analyze mode, feedback mode

o Implementation mode, task mode

Two different modes of activity: analytic mode and synthetic mode.


Question # 113: are dragged down from the title at the top of the screen.

o Pull Down Menus


Page # 132
o Main Menus

o Icons

o Buttons

Pull-down menus are dragged down from the title at the top of the screen, by moving the mouse pointer into the title par area
and pressing the button.

Question # 114: When you engaged in a conversation you are more attentive what the other person is saying. It is
called?

o Focused Attention
Page # 78
o Voluntary Attention

o Involuntary attention

o Divided Attention

Focused Attention: For example, when engaged in a conversation it is usual to attend to what the other person is saying.
Question # 115: Drive a vehicle while holding a conversation with a passenger is the example of
.

o Focused Attention

o Voluntary Attention

o Involuntary attention

o Divided Attention Page # 78

Divided Attention
As we said, we may be skilled at carrying on the conversation while intermittently observing what the person we want to talk
to is doing. When we attempt to attend to mire than one thing at a time, as in the above example, it is called divided attention.
Another example that is often used to illustrate this intentional phenomenon is being able to drive while holding a
conversation with a passenger.

Question # 116: GOMS stands for:

o Goals operation methods and selection rules


Page # 50
o Goals objects models and selection rules

o Goals operations methods and state rules

o Goals operations models and state rules

Question # 117: Which of the following is/are the main component(s) of color?

o Hue

o Intensity

o Saturation

o All of the given Page # 59

Hue: Hue is determined by the spectral wavelength of the light. Blues have short wavelength, greens medium and reds
long. Approximately 150 different hues can be discriminated by the average person.
Intensity: Intensity is the brightness of the color.
Saturation: Saturation is the amount of whiteness in the colors.
Question # 118: is a Usability Goal and refers to how easy a system is to remember how to
use, once learned.

o Learnability

o Memorability Page
# 32
o Utility

o None of the given


Memorability
It refers to how easy a system is to remember how to use, once learned. This is especially important for interactive
systems that are used infrequently.

Question # 119: A mouse button invites pushing by the way it is physically constrained in its plastic shell, is
an example of Design Principle.

o Visibility

o Affordance Page # 105

o Mapping

o None of the given

Affordance is a term used to refer to an attribute of an object that allows people to know how to use it. For example, a
mouse button invites pushing by the way it is physically constrained in its plastic shell.
Question # 120: Ali is working on a document called cv in MS word now wishes to make a copy of it on disk so
he can make some changes to the original. Which of the following process will help?

o Redo

o Archiving

o Undo

o Milestoning

Question # 121: When you try to log to yahoo. You wrongly enter password in capital letter, due to which error
message arise invalid username and password. Which of the following error message dialog will appear?

o Alerts

o Confirmation

o Feedback

o Delete.

Question # 122: With the help of Model, one can quantify (on the basis of Quantitative
Measurements) the human performance in using computer based system

o GOMS

o Human Processor model

o Quantum model

o Quality Model
Question # 123: Ethnography is a method that comes originally from anthropology and literally means

o Writing the culture Page #


173
o Social issues

o Ethical issues

o Ethics

Ethnography is a method that comes originally from anthropology and literally means “writing the culture”.

Question # 124: Physical objects are said to have affordance.

o Real Page # 105

o Perceived

o Logical

o Intuitive

Physical objects are said to have real affordances, like grasping, that are perceptually obvious and do not have to be
learned.

Question # 125: Which of these specialists is user centered?

o HCI Page # 21

o Software engineer

o Computing technology

o Web Designer
Human-computer interface specialists are user-centered and software engineers are system-centered.
CS408 - HUMAN COMPUTER INTERACTION
Oct
Solved MCQ(S) 21,2017
From Midterm Papers (1 TO 22 Lectures)
BY Arslan
V-U For Updated Files Visit Our Site : Www.VirtualUstaad.blogspot.com Updated.

MidTerm Papers Solved MCQS with Reference (1 to 22 lectures)


Question#1: The term mental model was first developed in the early 1640s by

o Johnson - Laird

o Donald Norman

o Kenneth Craik Page # 93

o John Corman

Question#2: In several keys must be pressed at once in order to enter a single character.

o QWERTY Keyboard

o Dvorak Keyboard

o Alphabetical keyboard

o Chord keyboards Page # 112

In chord keyboards several keys must be pressed at once in order to enter a single character. This is a bit like playing a
flute, where several keys must be pressed to produced with a small number of keys, few keys are required, so chord
keyboards can be very small, and many can be operated with just one hand.
Question#3: On my personal computer, I am listening a song with head phone. At same time, I am entering data in
MS word It means my system is:

o Uni-model

o Multi-model

o Mini-model

o Large-model

Question#4: What is the main strength of the Problem Space Framework as a model of human problem solving?

o It operates within the constraints of the human processing system Page # 91

o It explains what is involved in insight

o It allows ill-defined problems to be solved

o It allows poorly-defined problems to be solved

Question#5: Using icons on the desktop to represent operations is a type of constraint.

o Physical

o Logical

o Cultural Page # 106

o Graphical

Question#6: There is usually a attached to the top of a window identifying it to the user.

o Icon

o Title bar Page # 130

o Menu

o Button
Question#7: Input in human is mainly though the and output through the of the effectors.

o Senses, Rods

o Motor control, Senses

o Senses, Motor Control Page # 54

o Motor Control, Rods

Input in human is mainly though the senses and output through the motor control of the effectors. There are five
major senses:
• Sight
• Hearing
• Touch
• Taste
• Smell

Question#8: model is famous for risk analysis.

o Waterfall

o Spiral Page # 150

o RAD

o Incremental

For many years, the waterfall formed the basis of most software developments, but in 1988 Barry Boehm suggested the
spiral model of software development. Two features of the spiral model are immediately clear from figure: risk
analysis and prototyping.
Question#9: _aspect (s) gives us hints and tips about using and creating user interface idioms, like dialog
boxes and pushbuttons.

o Efficient.
o Strategic.
o Tactical.
o Reliable.

o Only A

o Only C Page # 143

o A and C

o B and D

Strategic aspect guides us to think about user interface idioms – in other words, the way in which the user and the idiom
interact.
Tactical aspects give us hints and tips about using and creating user interface idioms, like dialog boxes and pushbuttons.

Question#10: In order to develop any product, which major activity or activities should be undertaken from the
given list:

A. The designer must understand the requirements of the product


B. must develop the product.

o Only A

o Only B

o A and B Page # 148

o Not A and Not B

In order to develop any product, two major activities have to be undertaken:


The designer must understand the requirements of the product, and must develop the product.
Question#11: The persona's needs are fully satisfied by a primary interface refers to persona.

o Supplement Page # 159

o Negative

o Secondary

o Primary

Question#12: Suppose ali is writing an article on Cloud computing in windows mobile, which cognitive mode will
contribute to generate this idea.

o Experiential cognition

o Reflective cognition Page # 48

o Information cognition

o Execution cognition

Reflective cognition involves thinking, comparing, and decision-making. This kind of cognition is what leads to new
ideas and creativity. Examples include designing, learning, and writing a book.

Question#13: Primary interface serves the needs of the persona with the minor modification or addition refers to
persona.

o Primary

o Secondary Page # 159

o Supplement

o Negative
Question#14: As a designer you get usage patterns and the goals associated with users that drive the creation of
personas in the phase.

o Requirements definition

o Framework definition

o Modeling phase Page # 159

o Refinement

Question#15: minimize errors.

o Affordance

o Visibility

o Constraints Page # 106

o Affordance and Visibility

Constraints, on the other hand, are the inverse of affordances. They limit the way an object can be used.
Constraints can be used to avoid usage errors or minimize the information to be remembered.

Question#16: The color, which are directly opposite to each other are said to be.

o Complementary colors Page # 64

o Analogous colors

o Primary colors

o Secondary colors

Complementary colors are any two colors, which are directly opposite each other, such as red and green and red-purple
and yellow-green.
Question#17: The phase employs ethnographic field study techniques (observation and contextual
interviews) to provide qualitative data about potential and/or actual users of the product.

o Refinement

o Framework definition

o Modeling

o Research Page # 158

Question#18: Comparing design standards with guidelines, standards are usually _.

o Higher in authority and more limited in application

o Higher in authority and more general in application

o Lower in authority and more general in application

o Lower in authority and more limited in application


Question#19: There are various types of errors. Norman has categorized them into two main types.

o Mistakes, Error

o Self Blaming , Mistakes

o Error , Slips

o Mistakes, Slips Page # 100

Question#20: The transitions on the Interaction Framework;

o Are unimportant

o Simply link the whole system together

o Pass messages around the system

o Represent the translations required from one component to the next Page # 123

Question#21: Which of the following is proportional to the amplitude of the sound where the frequency remains
constant?

o Pitch

o Loudness Page # 70

o Timber

o Rhythm

Question#22: The eye is a mechanism for receiving light and transforming it into energy.

o Electrical energy Page # 55

o Heat energy

o Potential energy

o Kinetic energy

Vision begins with light. The eye is a mechanism for receiving light and transforming it into electrical energy. Light is
reflected from objects in the world and their image is focused upside down on the back of the eye. The receptors in
the eye transform it into electrical signals, which are passed to brain.
Question#23: WYSIWYG stands for .

o What you say is what you talk

o What you see is what you get Page # 36

o What you sing is what you think

o What you sow is what you ripe

WYSIWYG (what you see is what you get)

Question#24: Which of the following is haptic perception?

o Touch Page # 71

o Smell

o Taste

o Hear

The third sense is touch or haptic perception. Although this sense is oftern viewed as less important than sight or
hearing, imagine life without it. Touch provides us with vital information about our environment. It tells us when we
touch something hot or cold, and can therefore act as a warning. It also provides us with feedback when we attempt to
lfit and object.

Question#25: is required when someone is required to make a prediction about a


particular state of affairs.

o Image model

o Mental model Page # 94

o Initial model

o Spiral model

Mental models are usually constructed when we are required to make an inference or a prediction about a particular
state of affairs
Question#26: are individual and isolated regions within display that can be selected by the user to
invoke specific operations.

o Buttons Page # 133

o Edition

o Cursors

o Windows

Buttons are individual and isolated regions within display that can be selected by the user to invoke specific operations.

Question#27: In understanding the nature of usability: the way or approach in which the user and the idiom interact
is called .

o Strategic Page # 143

o Tactical

o Non- Technical

o Technical

Strategic aspect guides us to think about user interface idioms – in other words, the way in which the user and the idiom
interact.

Question#28: Unable to edit pdf document, is a type of constraints.

o Physical

o Logical Page # 106

o Cultural

o Update
Question#29: The appears in the center of the circle, and so there is the same distance to travel to any
of the selections.

o Main menus

o Icon

o Pointer Page # 132

o Title bar

Question#30: Which of the following is proportional to the amplitude of the sound; the frequency remains
constant?

o Pitch

o Loudness page # 70

o Timber

o None of these

Question#31: The human eye has approximately rods and _ cones per eye

o 120 million , 6 million Page # 56

o 190 million , 3 billion

o 150 billion , 6 million

o None of the given

The human eye has approximately 120 million rods 6 million cones per eye
Question#32: What are the three types of reasoning?

o Deductive, Productive and Inductive

o Abdicative, Inductive and Deductive Page # 89

o Inductive, Abdicative and Reproductive

o Productive, Reproductive and Deductive

Reasoning
Reasoning is the process by which we use the knowledge we have to draw conclusions or infer something new about
the domain of interest. There are a number of different types of reasoning:
• Deductive reasoning
• Inductive reasoning
• Abdicative reasoning

Question#33: Which of the following is not one of the goals of HCI?

o To produce usable systems

o To produce safe systems

o To produce non-functional systems Page # 30

o To improve effectiveness of the systems

The goals of HCI are to produce usable and safe systems, as well as functional systems. These goals can be summarized
as „to develop or improve the safety, utility, effectiveness, efficiency and usability of systems.

Question#34: Which of the following is considered as the most complex species?

o Humans Page # 18

o Computers

o Animals

o Birds

Human beings are the most interesting and fascinating specie on planet. They are the most complex living being on the
earth
Question#35: Totality of characteristics of an entity that bear on its ability to satisfy stated and implied needs are
called _.

o Quality Page # 40 Click here for more detail

o Effectiveness

o Standard

o Quantity

Quality is essential in all we do and particularly in our chosen field of work.


Quality has been defined by the International Standards Organization (ISO) as:
The totality of characteristics of an entity that bear on its ability to satisfy stated or implied needs.

Question#36: Which of the following is not true regarding "cones"?

o A type of receptor in eye

o more sensitive to light Page # 56

o different types of cones are sensitive to different wavelengths of light

o eye has approximately 6 million cones

Cones are the second type of receptor in the eye. They are less sensitive to light than the rods and can therefore tolerate
more light. There are three types of cone, each sensitive to a different wavelength of light. This allows color vision. The
eye has approximately 6 million cones, mainly concentrated on the fovea.

Question#37: Which of the following is a true statement?

o Human-computer interface specialists are user-centered and software engineers are


system centered Page # 21

o Human-computer interface specialists are system-centered and software engineers are user-centered

o Human-computer interface specialists and software engineers, both are system-centered.

o Human-computer interface specialists and software engineers, both are user-centered.

Software Engineering and HCI


There is a basic fundamental difference between the approaches taken by software engineers and human-computer
interaction specialists. Human-computer interface specialists are user-centered and software engineers are system-
centered.
Question#38: Which of the following is not true?

o Utility refers to the functionality of a system

o Usability is concerned with adding complexity to the system page # 30

o Usability is concerned with making systems easy to use

o Poorly designed computer system can be extremely annoying to users

Utility refers to the functionality of a system or, in other words, the things it can do. Improving effectiveness and
efficiency are self-evident and ubiquitous objectives. The promotion of safety in relation to computer systems is of
paramount importance in the design of safety-critical systems. Usability, a key concept in HCI, is concerned with
making systems easy to learn and easy to use. Poorly designed computer system can be extremely annoying to users, as
you can understand from above described incidents.

Question#39: Formal methods are used to represent

o Architecture aspects of software systems only

o Procedural aspects of software systems only

o Both Architecture and procedural aspects of software systems Page # 21

o None of the given

Formal methods have been developed to represent data, architectural, and procedural aspects of a software system

Question#40: Which interface system is based on the question / answer dialogue?

o Command Line Interfaces

o Query interfaces Page # 127 and 128

o Menus

o Natural Language Interfaces


Question#41: Choice of operations and services are offered on the screen through

o Pointers

o Toolbars

o Menus Page # 131

o None of the given

A menu presents a choice of operations or services that can be performed by the system at a given time.

Question#42: What will be the gulf of execution if the user is able to formulate and perform the actions easily;

o Smaller Page # 122

o Greater

o Balanced

o None of the Given

Question#43: Form-filling interfaces are used for

o data entry Page # 128

o data integration

o data manipulation

o data definition

Form-filling interfaces are used primarily for data entry but can be useful in data retrieval applications.

Question#44: Which of the following is flaw in waterfall model?

o requirements change overtime Page # 150

o maintenance can not be possible at the end

o testing should be done before requirements gathering

o All at the given

Flaws of waterfall model: One of the main flaws with this approach is that requirements change over time.
Question# 45: There are two aspects in which we can understand the nature of usability A. Strategic and Tactical.
B. Efficiency and Strategy C. Tactical and Reliability D. Efficiency and Reliability

o Only A Page # 143

o Only C

o A and D

o B and C

we can understand The nature of usability in two aspects


• Strategic
• Tactical

Question#46: Once the user input into system, the input language is translated into the language as operations to be
performed by the system.

o Task

o Core Page # 123

o Both

o None of the above

The only way the user can manipulate the machine is through the input, and so the task must be articulated within the
input language, the input language is translated into the core language as operations to be performed by the system.

Question# 47: User and system has its own unique language the language used by the system is called .

o Core Page # 123

o Task

o Both

o None of the above

The system‟s language is referred as the core language and the user‟s language is referred as the task language
Question#48: Which of the following is used to toggle between two states?

o check boxes

o Radio buttons Page # 133

o Toolbars

o menus

Radio Buttons
Buttons can also be used to toggle between two states

Question# 49: Which of the following is a text entry device?

o Keyboard Page # 110

o mouse

o monitor

o touch pad

Text entry devices: There are many text entry devices as given below:
Keyboard The most common method of entering information into the computer is through a
Keyboard QWERTY keyboard, Alphabetic keyboard, Dvorak Keyboard, Chord keyboards , Phone
pad and T9 entry

Question#50: "Mistakes" and "Slips" are two forms of

o Errors Page # 100

o goals

o evaluation

o objectives

There are various types of errors. Norman has categorized them into two main types, slips and mistakes:
Question# 51: The _ model should match the model.

o User, conceptual

o Conceptual, mental Page # 94 Click here for more Detail

o Mental, central

o Conceptual, central

Question#52: Which of the following is/are aspect(s) of human action?

o execution and evaluation Page # 99

o evaluation and efficiency

o efficiency and execution

o efficiency

Human action has two aspects, execution and evaluation.


Execution: involves doing something.
Evaluation: is the comparison of what happened in the world with what we wanted to happen

Question#53: Building things from user's perspective is called

o Functionality

o Usability page # 31

o Portability

o None of the given

Usability goals: To recap, usability in generally regarded as ensuring that interactive products are easy to learn,
effective to use, and enjoyable from user perspective.
Question#54: Certain factors that help us to interact with graphical based systems

o Indirect manipulation

o Direct manipulation Page # 86

o Both

o None of the given

GUI and direct manipulation interface are good environments for supporting this kind of learning

Question#55: If you are taking lecture and suddenly you hear music or voices from the other room. It is called?

o Focused attention

o Divided attention

o Voluntary attention

o Involuntary attention Page # 70

An everyday example of an involuntary act is being distracted from working when we can hear music or voices in the
next room

Question#56: Which of the following is a long-term individual difference?

o Age

o Sex Click here for more Detail

o Fatigue

o Color

Individual Differences Long term : Sex , Physical abilities , Intellectual abilities


Question#57: refers to how good a system at doing What it is supposed to do?

o Safety

o Usability

o Efficiency

o Effectiveness Page # 31

Effectiveness
It is a very general goal and refers to how good a system at doing what it is suppose to do.

Question#58: As a software designer, which aspects guide you to think about the Way in which the user and the
idiom interact?

o Tactical

o Manual

o Technical

o Strategic Page # 143

Strategic aspect guides us to think about user interface idioms – in other words, the way in which the user and the idiom
interact.

Question#59: Adeel renders financial services in “ABC” bank .He specialized in his field. What he is called as from
the following?

o Society of manufacturing engineer (SME)

o Subject matter expert (SME) Page # 168

o Small and medium enterprise (SME)

o Subject master engineering (SME)

Subject matter expert (SME)


Question#60: Scenario content and context are derived from information gathered during the phase and
analyzed during the phase (respectively)

o Modeling, implementation

o Modeling, Research

o Research, implementation

o Research, modeling Page # 100

Scenario content and context are derived from information gathered during the Research phase and analyzed during the
modeling phase.

Question#61: What is a semantic network?

o A model of long-term memory Page # 82

o A record of our memory of events

o The part of the brain which allows us to remember things

o A mechanism for improving memory

Long-term memory structure


There are two types of long-term memory: episodic memory and semantic memory.

Question#62: Which of the following is least likely to be revealed by a paper prototype?

o Your users don't know the term algorithm

o Toolbar buttons are too small to press Click here for more Detail

o The Help menu isn't in the right place

o Radio buttons are too small


Question#63: is a powerful, multipurpose design tool that helps overcome several problems
that currently plague the development of digital products.

o Scenario

o Persona Page # 187

o Prototype

o None

The persona is a powerful, multipurpose design tool that helps overcome several problems that currently plague the
development of digital products.

Question#64: The difference between the intentions and allowable actions is the:

o Gulf of Execution Page # 103

o Gulf of Evaluation

o Both of the above options

o None of these

The difference between the intentions and allowable actions is the gulf of execution.

Question#65: Attention is the process of selecting things to concentrate on, at a point in time, from the range of
.

o Possibilities available Page # 76

o Time Available

o None of these

Attention is the process of selecting things to concentrate on, at a point in time, from the range of possibilities available.

Question#66: The goals of HCI are:

o Usability and User Experience Page # 30

o Learn ability and Comfort

o Tasks and Goals

o All of the above options


Question#67: WYSIWYG stands for .

o Where you see is where you get

o What you see is what you get Page # 36

o When you see it when you get

WYSIWYG (what you see is what you get)

Question#68: is proportional to the amplitude of the sound.

o Pitch

o Loudness Page # 70

o Timber

o None of the given

Loudness is proportional to the amplitude of the sound; the frequency remains constant

Question#69: Which of the given statements correctly defines effectiveness in terms of one of the usability goals?

o It is a very general goal and refers to how good a system at doing what it is suppose to do.
Page # 31

o It refers to the way a system supports users in carrying out their tasks.

o It involves protecting the users from dangerous conditions

o It involves protecting the users from undesired situations

Effectiveness: It is a very general goal and refers to how good a system at doing what it is suppose to do.

Question#70: Which of the given statements correctly defines efficiency in terms of one of the usability goals?

o It is a very general goal and refers to how good a system at doing what it is suppose to do.

o It refers to the way a system supports users in carrying out their tasks. Page # 31

o It involves protecting the users from dangerous conditions

o It involves protecting the users from undesired situations

Efficiency: It refers to the way a system supports users in carrying out their tasks.
Question#71: User personas that are not primary or secondary are _ personas.

o Served

o Negative

o Customer

o Supplemental Page # 196

Supplemental personas
User personas that are not primary or secondary are supplemental personas

Question#72: Which of the following is the comparison of what happened in the world with what we wanted to
happen?

o Action

o Evaluation Page # 99

o Execution

o Implementation

Action Cycle: Human action has two aspects, execution and evaluation. Execution involves doing something.
Evaluation is the comparison of what happened in the world with what we wanted to happen.

Question#73: Research can tell you about what, how, many and why in rich, multivariate detail.

o Quantitative

o Qualitative Page # 166

o SME

o None of these

Qualitative research can tell you about what, how and why in rich, multivariate detail.
Question#74: Requirements specification involves:

A. Capturing the functional requirements of the system architecture


B. Eliciting information about the Work domain from the customer.

o Only A

o Only B Click here for more Detail

o Both A and B

o Not A and Not B

Question#75: What are the Most common things you do with the product is a type of .

o Goal- oriented question

o System - oriented question Page # 182

o Workflow- oriented question

o Attitude-oriented question

System-oriented questions
Function
What are the most common things you do with the product?

Question#76: What do you procrastinate on? is type of

o Priorities

o Avoidance Page # 183

o Exceptions

o Preference

Avoidance
What would you prefer not to do?
What do you procrastinate on?
Question#77: Cognitive process involves encoding and recalling knowledge and acting appropriately

o Attention

o Reduction

o Memory Click here for more Detail and also Read PPT slides

o Encoding

Memory : Involves encoding and recalling knowledge and acting appropriately

Question#78: During usage and workflow patterns discovered through analysis of the field research and
interviews are synthesized into domain and user models

o Research

o Modeling Page # 159

o Framework

o Interview

Modeling: During the modeling phase, usage and workflow patterns discovered through analysis of the field research
and interviews are synthesized into domain and user models.

Question#79: Transactional sites that are used for a significant part of an employee‟s job should be considered full
applications.

o Sovereign Page # 30

o Transient

o Temporary

o None of the given

Transactional sites that are used for a significant part of an employee's job should be considered full sovereign
applications.
Question#80: is the process of selecting things to concentrate on, at a point in time, from the range
of possibilities available.

o Perception and recognition

o Attention Page # 76

o Learning

o None of these

Attention is the process of selecting things to concentrate on, at a point in time, from the range of possibilities available.

Question#81: HCI deals with .

o Design of interactive system only

o Evaluation of interactive system only

o Implementation of interactive system only

o All of the given choices Click Here For More Detail

Question#82: When you engaged in a conversation you are more attentive what the other person is saying. It is
called?

o Focused Attention Page # 78

o Voluntary Attention

o Involuntary attention

o Divided Attention

Focused Attention
For example, when engaged in a conversation it is usual to attend to what the other person is saying.
Question#83: Drive a vehicle while holding a conversation with a passenger is the example of _.

o Focused Attention

o Voluntary Attention

o Involuntary attention

o Divided Attention Page # 78

Divided Attention
As we said, we may be skilled at carrying on the conversation while intermittently observing what the person we want
to talk to is doing. When we attempt to attend to mire than one thing at a time, as in the above example, it is called
divided attention. Another example that is often used to illustrate this intentional phenomenon is being able to drive
while holding a conversation with a passenger.

Question#84: minimizes errors.

o Affordance

o Constraints Page # 106

o Visibility

o Affordance and Visibility

Question#85: are dragged down from the title at the top of the screen.

o Pull Down Menus Page # 132

o Main Menus

o Icons

o Buttons

Pull-down menus are dragged down from the title at the top of the screen, by moving the mouse pointer into the title
par area and pressing the button.
Question#86: aspect (s) gives us hints and tips about using and creating user interface idioms, like dialog
boxes and pushbuttons.

o Efficient

o Strategic

o Tactical Page # 143

o Reliable

Tactical aspects give us hints and tips about using and creating user interface idioms, like dialog boxes and
pushbuttons.

Question#87: The Usability Engineering lifecycle was proposed by .

o Hartson

o Webster

o Barry Boehm

o Deborah Mayhew Page # 153

The Usability Engineering lifecycle was proposed by Deborah Mayhew in 1999.

Question#88: Ali is looking at how interface designers went about their work. He identified two different modes of
activity: one is _ and other is __.

o Analytic mode, synthetic mode Page # 153

o Evaluation mode, testing mode

o Analyze mode, feedback mode

o Implementation mode, task mode

Two different modes of activity: analytic mode and synthetic mode.


Question#89: Alia is used to describe design problem /solution by that is an imagined or projected
sequence of events, especially any of several detailed plans or possibilities

o Persona

o Scenario Click Here For More Detail

o Flowchart

o Brainstorming

Question#90: are individual and isolated regions within display that can be selected by the user to
invoke specific operations.

o Buttons Page # 133

o Pointers

o Menus

o Windows

Buttons are individual and isolated regions within display that can be selected by the user to invoke specific operations.

Question#91: Around 63% of software projects exceed their cost estimates. The top four reasons for
This are:
A– Frequent requests for changes from users
B– Overlooked tasks
C– Users' lack of understanding of their own requirements
D– Insufficient user-analyst communication and understanding

o Only A

o Only B

o Only D

o ABC&D Page # 27

Around 63% of software projects exceed their cost estimates. The top four reasons for
this are:
– Frequent requests for changes from users
– Overlooked tasks
– Users' lack of understanding of their own requirements
– Insufficient user-analyst communication and understanding
Question#92: of the potential sales from a site are lost because people cannot find the item they are
looking for

o 50% Page # 141

o 40%

o 30%

o 10%

50% of the potential sales from a site are lost because people cannot find the item they are looking for

Question#93: Human eye is very sensitive to

o Air

o Smoke

o Light Page # 55 and 56

o Sand

Question#94: Top of the web page contain the

o Menu

o Title bar Click here For More Detail

o Home icon

o Back button

The title bar is a bar located at the top of a window or a dialog box that displays the name of the window or software
program being used.
Question#95: You can load a VCR tape the right way because of .

o Physical constraints Page # 106

o Logical constraints

o Cultural constraints

o None of these

Physical constraints: Physical constraints refer to the way physical objects restrict the movement of things. For
example, the way a external disk can be placed into a disk drive is physically constrained by its shape and size, so that it
can be inserted in only one way. Likewise, keys on a pad can usually be pressed in only one way.

Question#96: are unintentional while occur through conscious deliberation


(respectively).

o Slips, mistakes Page # 100

o Errors, slips

o Mistakes, errors

o Mistakes, slips

Slips
Slips are unintentional. They happen by accident, such as making typos by pressing the wrong key or selecting wrong
menu item by overshooting. The most frequent errors are slips, especially in well-learned behavior.

Mistakes
Mistakes occur through conscious deliberation. An incorrect action is taken based on an incorrect decision. For
example, trying to throw the icon of the hard disk into the wastebasket, in the desktop metaphor, as a way of removing
all existing files from the disk is a mistake. A menu option to erase the disk is appropriate action.

Question#97: What is the main strength of the Problem Space Framework as a model of human problem solving?

o It operates within the constraints of the human processing system Page # 91

o It explains what is involved in insight

o It allows ill-defined problems to be solved

o None of these
Question#98: Over a short period of time, we find it easier to remember the string of numbers "404 894 6743"
because:

o Numbers are easier to remember than arbitrary characters.

o The grouping of the numbers is significant Page # 81

o Ten numbers is not that many to have to remember from working memory.

o None of these

Question#99: Interpretation inquiry, according to Beyer and Holtzblatt, is based on a master apprentice model of
learning.

o True

o False Page # 176

Contextual inquiry: Contextual inquiry, according to Beyer and Holtzblatt, is based on a master-apprentice model of
learning: observing and asking questions of the users as if she is the master craftsman and he interviews the new
apprentice.

Question#100: The persona is not an actual user of the product, but is indirectly affected by it and its use refers to
persona

o Primary

o Secondary

o Served Page # 159

o Negative

Served: the persona is not an actual user of the product, but is indirectly affected by it and its use

Question# 101: Which of the following is not a secondary color?

o Green

o Orange

o Purple

o Blue Page # 63

Secondary Colors: These are the colors formed by mixing the primary colors. Green, orange and purple
Question#102: Which of the following requires less cognitive effort than others?

o Listening Page # 87

o Speaking

o Hearing

o None of the given

Listening require less cognitive effort than reading or speaking.

Question#103: We are deficient in our development , not in our development


(respectively).

o Process, Tools Page # 23

o Tools, Process

o Tools, Methodology

o None of these

We are deficient in our development process, not in our development tools.

Question#104: The command line interface is used because:

o It is easy to understand

o It is demanded by DOS

o It is offered by UNIX systems

o It is powerful and flexible Page # 127

Command line interface are powerful in that they offer direct access to system functionality, and can be combined to
apply a number of tools to the same data. They are also flexible: the command often has a number of options or
parameters that will vary its behavior in some way, and it can be applied to many objects at once, making it useful for
repetitive tasks.
Question#105: Which of the following is concerned primarily with understanding human behavior and the mental
processes that underlie it?

o Psychology Page # 43

o Sociology

o Statistics

o Computer Science

Psychology is concerned primarily with understanding human behavior and the mental processes that underlie it.

Question#106: plays a role to bridge up the gape between the interfaces of machines and
human understanding.

o Human

o Computer

o Human Computer Interaction Page # 14

o None of these

HCI plays a role to bridge up the gape between the interfaces of machines and human understanding that we have
seen in the previous examples.

Question#107: The persona‟s needs are sufficiently unique to require a distinct interface form and behavior is
type of Persona.

o Primary Page # 159

o Secondary

o Supplement

o Negative

Primary: the persona’s needs are sufficiently unique to require a distinct interface form and behavior
Question#108: A is usually a collection of icons those are reminiscent of the purpose of the
various modes.

o Button

o Pointer

o Palette Page # 133

o Title bar

A palette is usually a collection of icons that are reminiscent of the purpose of the various modes.

Question#109: During phase, usage and workflow patterns discovered through .

o Modeling, analysis Page # 159

o Analysis, modeling

o Testing, modeling

o Testing, Analysis

Modeling: During the modeling phase, usage and workflow patterns discovered through analysis of the field research
and interviews are synthesized into domain and user models.

Question#110: The Star lifecycle was proposed by .

o Deborah Mayhew

o Webster

o Barry Boehm

o Hartson Page # 152

In 1989, the Star lifecycle model was proposed by Hartson and Hix.
Question#111: Waterfall model is basically a model in which each step must be completed before the
next step can be started.

o Incremental

o Linear Page # 149

o Iterative

o Analytical

The waterfall lifecycle was the first model generally known in software engineering and forms the basis of many
lifecycle in use today. This is basically a linear model in which each step must be completed before the next step can
be started.

Question#112: What do you enjoy most about your job (or lifestyle) is an example of .

o Avoidance

o Motivation Page # 183

o Exceptions

o Attitude-oriented questions

Motivation
What do you enjoy most about your job (or lifestyle)?
What do you always tackle first?

Question#113: represents the Early-phase of ethnographic interviews.

o Clarify user roles and behaviors

o Confirm patterns of use

o Clarifying questions

o Focused on domain knowledge Page # 181

Early-phase
• Exploratory
• Focused on domain knowledge
• Open-ended questions
114: There can only be one persona per interface for a product.

o Primary Page # 196

o Secondary

o Supplemental

o Customer

Primary personas represent the primary target for the design of an interface. There can be only one primary persona per
interface for a product.

Question#115: What are the most common things you do with the product is a type of .

o Goal-oriented question.

o System-oriented question. Page # 182

o Workflow-oriented question.

o Attitude-oriented question.

System-oriented questions
Function: What are the most common things you do with the product?

Question#116: represent the user‟s expectations of the tangible outcomes of using a specific product.

o Non-user goals

o End goals Page # 192

o Experience goals

o Life goals

End goals represent the user‟s expectations of the tangible outcomes of using specific product.
Question#117: personas address the needs of customers, not end users.

o Served

o Negative

o Customer Page # 197

o Supplemental

Customer personas address the needs of customers, not end users. Typically, customer personas are treated like
secondary personas.

Question#118: Persona is not context specific, so it can be reused easily.

o True

o False Page # 189

Personas and reuse


– Personas context-specific
• Cannot be reused across products
Slide No. 22 Page # 21/61

Question#119: The size and shape of a compact disk, is a type of constraint.

o Physical Page # 106

o Logical

o Cultural

o None of these

Physical constraints
Physical constraints refer to the way physical objects restrict the movement of things. For example, the way a external
disk can be placed into a disk drive is physically constrained by its shape and size, so that it can be inserted in only one
way. Likewise, keys on a pad can usually be pressed in only one way.
Question#120: Which of the following device can not be useful for a visually impaired person?

o a typical keyboard

o a typical monitor

o a typical speaker

o a typical processor

Monitors are useless to the visually impaired or blind

person. Question#121: The gulf of execution refers to _.

o The user‟s difficulty in understanding the feedback from the system

o The system‟s difficulty in presenting information in the output language

o The system‟s difficulty in converting an input expression into the correct system state transition

o The user’s difficulty in formulating and articulating an intention to the system

Question#122: Which of the following is not one of the primary colors?

o Red

o Yellow

o Blue

o Green Page # 62

Primary Colors
In traditional color theory, these are the 3 pigment colors that cannot be mixed or formed by any combination of other
colors. All other colors are derived from these 3 hues
PRIMARY COLORS
Red, yellow and blue
Question#123: Learnability, flexibility and robustness are three main usability principles that can be considered as
general headings for standards and guidelines generation. Which of the following are also high level usability categories
that can guide standards and guidelines generation?

(i) Effectiveness
(ii) Efficiency
(iii) Fault tolerance
(iv)Satisfaction

Select correct option:

o (i) & (ii) Page # 31

o (i), (ii) & (iv)

o (ii) & (iii)

o (ii) & (iv)

usability is broken down into the following goals:


• Effective to use (effectiveness)
• Efficient to use (efficiency)
• Safe to use(safety)
• Have good utility (utility)
• Easy to learn (learnability)
• Easy to remember how to use (memorability)

Question#124: Human beings interact with outside world, using their

o input channels

o output channels

o sense of sight

o All of the given Page # 54

A person‟s interaction with the outside world occurs through information being received and sent: input and output.
Sight may be used primarily in receiving information from the computer, but it can also be used to provide information
to the computer, for example by fixating on a particular screen point when using an eye gaze system.
Question#125: Visually impaired persons can interact with outside world using their

o Sense of sight

o Sense of hearing

o Both sense of touch and sense of hearing ( Blind Person can interact with sense of touch
and hearing)

o Sense of touch

Question#126: refers to the relationship between controls and their effects in the world.

o Visibility

o Affordance

o Mapping Page # 107

o None of the given

Mapping
This refers to the relationship between controls and their effects in the world

Question#127: is a very general goal of Usability and refers to how good a system at doing what it is
suppose to do.

o Effectiveness Page # 31

o Efficiency

o Utility

o None of the given

Question#128: is what goes on in out heads when we carry out our everyday activities?

o Cognition Page # 47

o Learnability

o Memorability

o None of the given


Question#129: is the process by which we use the knowledge we have to draw conclusions or infer
something new about the domain of interest.

o Decision Making

o Reasoning Page # 89

o Problem Solving

o None of the given

Question#130: research helps us understand the domain, context and constraints of a product in
different, more useful ways than _ research do.

o Qualitative, Quantitative Page # 167

o Quantitative, Qualitative

o Qualitative, Deductive

o None of them

Question#131: In an organization individuals may keep their own records, or there may be local gurus. This
statement concerns with .

o Paper work and computer work

o Spatial and temporal organizations

o Organizational memory Page # 176

o None of these

Organizational memory
Formal documents are not the only way in which things are remembered within an organization. Individuals may keep
their own records, or there maybe local gurus.
Question#132: Using icons on the desktop to represent operations is a type of constraint.

o Physical

o Logical

o Cultural Page # 106

o None of these

Question#133: is a term used to refer to an attribute of an object that allows people to know how to
use it.

o Visibility

o Affordance Page # 105

o Constraint

o None of these

Question#134: minimizes errors.

o Affordance

o Visibility

o Constraints Page # 106

o None of these

Question#135: Models are used in design to:

o Generate the design

o Evaluate the design

o Generate and evaluate the design Click here for more Detail

o None of the given


Question#136: Which are the most significant senses for the average person when it comes to interacting with a
computer?

o Sight and hearing

o Hearing, touch and smell

o Hearing and touch

o Sight, hearing and touch Click here for more Detail

The major senses in human interaction are vision, hearing and touch.

Question#137: refers to the way a system supports users in carrying out their tasks.

o Efficiency Page # 31

o Effectiveness

o Utility

o None of the given

Efficiency
It refers to the way a system supports users in carrying out their tasks.

Question#138: are GUIs that consists of electronic counterparts to physical objects in the real world to
match the knowledge requirements of users.

o User Interaction Models

o Conceptual Models

o Interface Metaphors Page # 51

o None of the given

Interface metaphors are (these are GUIs that consists of electronic counterparts to physical objects in the real world) to
match the knowledge requirements of users.
Question#139: language tends to be grammatical while language is often Ungrammatical.

o Written, spoken Page # 87

o Spoken, written

o Verbal, non-verbal

o None of the given

Written language tends to be grammatical while spoken language is often ungrammatical.

Question#140: Aspect gives us hints and tips about using and creating user interface idioms.

o Strategic

o Tactical Page # 143

o Operational

o None of the given choices

Strategic aspect guides us to think about user interface idioms – in other words, the way in which the user and the idiom
interact. Tactical aspects give us hints and tips about using and creating user interface idioms, like dialog boxes and
pushbuttons.

Question#141: Faisal is looking at how interface designers went about their work. He identified two different
modes of activity: one is and other is _ .

o Analytic mode, synthetic mode Page # 153

o Evaluation mode, testing mode

o Analyze mode, feedback mode

o Implementation mode, task mode

Two different modes of activity: analytic mode and synthetic mode.


Question#142: are dragged down from the title at the top of the screen.

o Pull Down Menus Page # 132

o Main Menus

o Icons

o Buttons

Pull-down menus are dragged down from the title at the top of the screen, by moving the mouse pointer into the title par
area and pressing the button.

Question#143: When you engaged in a conversation you are more attentive what the other person is saying. It is
called?

o Focused Attention Page # 78

o Voluntary Attention

o Involuntary attention

o Divided Attention

Focused Attention: For example, when engaged in a conversation it is usual to attend to what the other person is
saying.

Question#144: Drive a vehicle while holding a conversation with a passenger is the example of
.

o Focused Attention

o Voluntary Attention

o Involuntary attention

o Divided Attention Page # 78

Divided Attention
As we said, we may be skilled at carrying on the conversation while intermittently observing what the person we want
to talk to is doing. When we attempt to attend to mire than one thing at a time, as in the above example, it is called
divided attention. Another example that is often used to illustrate this intentional phenomenon is being able to drive
while holding a conversation with a passenger.
Question#145: GOMS stands for:

o Goals operation methods and selection rules Page # 50

o Goals objects models and selection rules

o Goals operations methods and state rules

o Goals operations models and state rules

Question#146: Which of the following is/are the main component(s) of color?

o Hue

o Intensity

o Saturation

o All of the given Page # 59

Hue: Hue is determined by the spectral wavelength of the light. Blues have short wavelength, greens medium and reds
long. Approximately 150 different hues can be discriminated by the average person.
Intensity: Intensity is the brightness of the color.
Saturation: Saturation is the amount of whiteness in the colors.

Question#147: is a Usability Goal and refers to how easy a system is to remember how to use, once
learned.

o Learnability

o Memorability Page # 32

o Utility

o None of the given

Memorability
It refers to how easy a system is to remember how to use, once learned. This is especially important for interactive
systems that are used infrequently.
Question#148: A mouse button invites pushing by the way it is physically constrained in its plastic shell, is an
example of Design Principle.

o Visibility

o Affordance Page # 105

o Mapping

o None of the given

Affordance is a term used to refer to an attribute of an object that allows people to know how to use it. For example, a
mouse button invites pushing by the way it is physically constrained in its plastic shell.

Question#149: Ali is working on a document called cv in MS word now wishes to make a copy of it on disk so he
can make some changes to the original. Which of the following process will help?

o Redo

o Archiving Page # 321

o Undo

o Milestoning

Question#150: When you try to log to yahoo. You wrongly enter password in capital letter, due to which error
message arise invalid username and password. Which of the following error message dialog will appear?

o Alerts Page # 387

o Confirmation

o Feedback

o Delete.
Question#151: With the help of Model, one can quantify (on the basis of Quantitative Measurements) the
human performance in using computer based system

o GOMS Page # 50

o Human Processor model

o Quantum model

o Quality Model

Question#152: Ethnography is a method that comes originally from anthropology and literally means

o Writing the culture Page # 173

o Social issues

o Ethical issues

o Ethics

Ethnography is a method that comes originally from anthropology and literally means “writing the culture”.

Question#153: Physical objects are said to have affordance.

o Real Page # 105

o Perceived

o Logical

o Intuitive

Physical objects are said to have real affordances, like grasping, that are perceptually obvious and do not have to be
learned.

Note: Give me a feedback and your Suggestion also If you find any mistake in mcqz plz inform
me Viva Contact us Page on our Site. And tell me your answer with references.

For More Solved Papers By Arslan Visit Our Website :


Www.VirtualUstaad.blogspot.com
Question No: 167 (Marks: 1)

You might also like